Jump to content

Wikipedia:Reference desk/Science: Difference between revisions

From Wikipedia, the free encyclopedia
Content deleted Content added
→‎Elder people: some ideas on where to start research.
Oxio-a (talk | contribs)
No edit summary
Line 573: Line 573:


:There are dozens of different goal-directed and goal-oriented pedagogy programs. You're going to have to be a bit more specific. If you are interested in neural substrates in general, see the article [[Cognitive neuroscience]], which discusses them. --[[User:Jayron32|<font style="color:#000099">Jayron</font>]]'''''[[User talk:Jayron32|<font style="color:#009900">32</font>]]''''' 04:25, 7 June 2010 (UTC)
:There are dozens of different goal-directed and goal-oriented pedagogy programs. You're going to have to be a bit more specific. If you are interested in neural substrates in general, see the article [[Cognitive neuroscience]], which discusses them. --[[User:Jayron32|<font style="color:#000099">Jayron</font>]]'''''[[User talk:Jayron32|<font style="color:#009900">32</font>]]''''' 04:25, 7 June 2010 (UTC)

Well, within the context of contingency learning and extinction.


== CNG engines and simple engines ==
== CNG engines and simple engines ==

Revision as of 04:35, 7 June 2010

Welcome to the science section
of the Wikipedia reference desk.
Select a section:
Want a faster answer?

Main page: Help searching Wikipedia

   

How can I get my question answered?

  • Select the section of the desk that best fits the general topic of your question (see the navigation column to the right).
  • Post your question to only one section, providing a short header that gives the topic of your question.
  • Type '~~~~' (that is, four tilde characters) at the end – this signs and dates your contribution so we know who wrote what and when.
  • Don't post personal contact information – it will be removed. Any answers will be provided here.
  • Please be as specific as possible, and include all relevant context – the usefulness of answers may depend on the context.
  • Note:
    • We don't answer (and may remove) questions that require medical diagnosis or legal advice.
    • We don't answer requests for opinions, predictions or debate.
    • We don't do your homework for you, though we'll help you past the stuck point.
    • We don't conduct original research or provide a free source of ideas, but we'll help you find information you need.



How do I answer a question?

Main page: Wikipedia:Reference desk/Guidelines

  • The best answers address the question directly, and back up facts with wikilinks and links to sources. Do not edit others' comments and do not give any medical or legal advice.
See also:



June 2

Decaffeinated coffee acclimatisation

Can decaffeinated coffee still have an effect, whether placebo or directly? I mean, clearly it contains nothing to compete with adenosine so it can't work the same way, and I'm skeptical, but decaffeinated coffee definitely has an effect on my personal sleeping habits. Why would this be? How? Regards, --—Cyclonenim | Chat  00:36, 2 June 2010 (UTC)[reply]

One thing to keep in mind is that decaffeinated coffee may still contain clinically significant amounts of caffeine. I suspect this is especially true for higher-quality blends, where the priority is flavor rather than removing all the caffeine you possibly can, but I don't really know that. --Trovatore (talk) 00:46, 2 June 2010 (UTC)[reply]
Decaffeinated does not mean caffeine free. --Chemicalinterest (talk) 00:59, 2 June 2010 (UTC)[reply]
I would be quite surprised if there wasn't a placebo effect, but generally that would require the drinker to think they were drinking regular coffee. I'm guessing that isn't the case with you, so it is unlikely to be placebo. It could just be confirmation bias, though. Alternatively, Trovatore may be right and the small amounts of caffeine still in the coffee are having the effect. --Tango (talk) 01:08, 2 June 2010 (UTC)[reply]
Our article on decaffeination makes reference to an "international standard" that 97% of the caffeine must have been removed, and to an "EU standard" that the beans must be 99.9% caffeine-free. On the other hand, one of the references is to this Science Daily article, which implies that decaf may have as much as 20% of the caffeine of regular coffee.
These facts seem somewhat difficult to reconcile. One possibility is that the coffee referred to by ScienceDaily was not compliant with either of these standards. Another is that beans consisting of 0.1% caffeine by weight (how much is in normal beans?) still produce coffee with 20% the caffeine of regular (maybe in part because you need more coffee grounds to get the same concentration of flavor chemicals?). --Trovatore (talk) 01:40, 2 June 2010 (UTC)[reply]
The amount of caffeine is strongly influenced by brewing method, so it's difficult to make good comparisons. Ariel. (talk) 03:40, 2 June 2010 (UTC)[reply]
Original research: When I drink a carbonated beverage, I instantly feel a buzz whether or not it has caffeine; I have to think this is some sort of placebo effect my brain is pulling on me (unless it's the carbonation itself). -RunningOnBrains(talk) 05:25, 2 June 2010 (UTC)[reply]
I'd guess it's the sugar giving you the buzz. --Tango (talk) 11:00, 2 June 2010 (UTC)[reply]
Can you even get a placebo effect if you know it is a placebo? Googlemeister (talk) 13:47, 2 June 2010 (UTC)[reply]
Maybe with a bit of doublethink! --Tango (talk) 16:16, 2 June 2010 (UTC)[reply]

Still has caffeine, but maybe similar chemicals (take one or two methyl groups off) also give insomnia. That rooibos tea and chicory root is caffeine free but still has other stimulants. Polypipe Wrangler (talk) 13:40, 2 June 2010 (UTC)[reply]

Thanks for the responses :) Regards, --—Cyclonenim | Chat  17:05, 2 June 2010 (UTC)[reply]

Different kinds of cold?

Why is it that I could stand cold weather or the temperature in Baguio City but shiver when near an air conditioning unit? Do outside weather and air conditioning cools us differently?--Lenticel (talk) 02:12, 2 June 2010 (UTC)[reply]

For one thing, the air conditioner is probably blowing the air at you. Moving air will almost always "feel" colder than standing air. (See Wind chill for an extreme example of this). Buddy431 (talk) 02:39, 2 June 2010 (UTC)[reply]
One thought is that usually when you're outside you're not just hanging out, you're doing something even if it's only walking from place to place. Inside you're liking to be sitting in a chair and not exerting yourself. APL (talk) 04:17, 2 June 2010 (UTC)[reply]
Unfortunately, Thermoception is not a very good article, but that may be what you're looking for. We don't typically feel absolute hot and cold, just relative temperatures, so if you put one hand in hot water, another in cold water, and then put both in lukewarm water, the lukewarm water would feel hot to the hand that was in cold water, and vice versa. If you're air conditioning is much cooler. Also, you likely wear warm clothes outside in the cold, but not inside in your air conditioning. Furthermore, forgive a little teasing, but "cold" in Baguio City (record low: 46 °F (7.8 °C)) isn't exactly the same as the "cold" I know (record low: −24 °F (−31 °C)) -RunningOnBrains(talk) 05:22, 2 June 2010 (UTC)[reply]
Dude, I live in the hot and humid tropics, so that's already cold :)--Lenticel (talk) 00:55, 3 June 2010 (UTC)[reply]
46 °F (7.8 °C)!?? Haha. I still wear shorts and a t-shirt in that kind of weather if there's not too windy! (admittedly, some neighbors think I'm crazy.)
Actually, the air coming out of your air conditioner could literally be colder than that. (Sure, your thermostat is set to something reasonable, but to achieve that it needs to mix some really cold air into your hot home.) APL (talk) 06:12, 2 June 2010 (UTC)[reply]
Air conditioners don't just cool the air, they also dry it, which makes it cool your skin more via evaporation. Since Baguio City, at an altitude of a mile above sea level in the Philippines, must be one of the most humid places on Earth, it can make a big difference. Looie496 (talk) 16:37, 2 June 2010 (UTC)[reply]
Humidity. Ah that's must be it! I mean the air is still humid even during the El Nino season. --Lenticel (talk) 01:22, 3 June 2010 (UTC)[reply]

"Dirty dozen" fruits and vegetables

CNN.com has an article on fruits and vegetables that contain pesticides even after being washed. The article has a "dirty dozen" list and a "clean 15" list of produce. What is confusing is that while potatoes and lettuce are on the dirty list, sweet potatoes and cabbage are on the clean list. Why? There seems to be no simple rules of thumb that can help you figure out for what produce you should go organic. --173.49.15.136 (talk) 02:40, 2 June 2010 (UTC)[reply]

My guess would be the susceptibility of the plant to damaging insect invasion leads to the use of insecticide. Bus stop (talk) 03:05, 2 June 2010 (UTC)[reply]
For sweet potatos, they may be assuming that people aren't going to eat the skin. That's what appears to separaten the clean vs. dirty foods to a large extent: whether people eat the outside layer. I like how they emphasize the number of different types of pesticides in the different types of produce. Wouldn't make more sense to look at the total amounts of pesticides, rather than the variety? Buddy431 (talk) 03:17, 2 June 2010 (UTC)[reply]
"Give me spots on my apples; But leave me the birds and the bees" (Joni MitchellBig Yellow Taxi) Bus stop (talk) 03:22, 2 June 2010 (UTC)[reply]
Who is applying so many different types of pesticide on the one foodstuff? I find it hard to believe that some foodstuffs contain up to 67 different pesticides. Caesar's Daddy (talk) 07:32, 2 June 2010 (UTC)[reply]
A lot of it's probably contamination from other types of produce grown nearby; a good wind and you've got on your plant whatever your neighbor put on his. Still, 47-67 different pesticides does seem pretty excessive. Is there anyone here with experience in this type of thing who can explain how, and how many, pesticides are typically applied to different types of food? Buddy431 (talk) 16:34, 2 June 2010 (UTC)[reply]
A "single" pesticide, may actually be a combination of multiple chemicals. Like how with roundup the article mentions how roundup itself is pretty safe, but it's extensive list of additives are the problem. It doesn't say how many it has, but if other pesticides are like that, that could easily increase the number. Just a guess. Ariel. (talk) 18:53, 2 June 2010 (UTC)[reply]

Terraformation of Mars

In this picture, they show an artists conception of the terraformation of mars. In stages 3 and 4, they basically just add more water, making the oceans bigger. What exactly is the advantage of that? Wouldn't it be better to have more land than oceans? ScienceApe (talk) 03:42, 2 June 2010 (UTC)[reply]

Perhaps because oceans might help regulate the Martian climate better Ocean#Climate_effects.--Lenticel (talk) 03:47, 2 June 2010 (UTC)[reply]
Water is also the most important greenhouse gas on Earth, and given how cold Mars is, covering a larger surface area with standing water might contribute to getting more water into the air to help support this warming. (Of course to get an ocean in the first place you already have to add something to the atmosphere to trap heat and keep the water from simply freezing.) Dragons flight (talk) 03:53, 2 June 2010 (UTC)[reply]
There are several reasons to have an ocean. First, the above-mentioned greenhouse effect. Second, water has a very high specific heat capacity, and because it is mostly transparent, serves as a giant heat sink, which keeps the earth's temperatures from fluctuating too much over the course of a day (currently, martian temperatures can change almost 100 °C (180 °F) in a day). Thirdly, if you hope to terraform a planet, and introduce large amounts of life, you're going to need large amounts of water.
Unfortunately, terraforming Mars is unlikely to work. Mars' low mass means that, unlike on earth, important molecules such as nitrogen, oxygen, and, yes, water vapor can escape its gravity directly into space over time. Thus, we would need to constantly replenish any atmosphere we created there. This is of course ignoring the HUGE question of "where the heck to we get a planet's supply of water and oxygen from in the first place?" :-D -RunningOnBrains(talk) 05:08, 2 June 2010 (UTC)[reply]
Atmosphere loss is only significant on a geological time scale. Unless the teraforming is planned to take billions of years, replenishing the atmosphere would be trivial compared to the task of creating one in the first place. --Bowlhover (talk) 08:11, 2 June 2010 (UTC)[reply]
To get the water there in the first place (assuming we don't find enough frozen down under the surface already) would probably require bombarding the planet with icy asteroids - no small feat! The whole issue is really about how to warm the planet up - doing that would make it possible to drive water and various gasses out of the soil. There is a lot of frozen CO2 at the poles, melting the icecaps could liberate that and cause enough of a global warming effect to heat up everything else. Once things are warm and there is a denser atmosphere (albeit mostly of CO2) then you'd want to add photosynthesis to the mix to turn CO2 into oxygen. But this is a crazily difficult project. Nobody should ever underestimate the difficulties. SteveBaker (talk) 11:53, 2 June 2010 (UTC)[reply]
Indeed, it is beyond our ability to really conceive of how it would be done. It's not going to be done in the next 25 years, and as far as technology is concerned any predictions more than 25 years ahead are little more than guesses. --Tango (talk) 16:18, 2 June 2010 (UTC)[reply]
There are arguments that the bulk of Mars' former atmosphere has not been lost to space, but rather has become locked up in its rocks because, unlike on the Earth, it does not get recycled by tectonic activity and active vulcanism. Theorists such as Martyn J. Fogg (a personal acquaintance) have proposed releasing significant quantities from appropriate strata by large scale thermonuclear engineering (i.e. bury huge H-bombs and set 'em off, preferably from a distance). Such released gasses might, by positive global warming feedback mechanisms similar to those we ourselves are arguably experiencing, cause further atmospheric evolution and result in a useful atmosphere quite rapidly, which might persist on a timescale of the order of 100,000 years or more before further replenishment became necessary. 87.81.230.195 (talk) 22:09, 2 June 2010 (UTC)[reply]
I was under the impression that lack of a magnetosphere was a (the?) reason Mars doesn't have an atmosphere. From the magnetosphere article: "Earth’s magnetosphere provides protection, without which life as we know it could not survive. Mars, with little or no magnetic field is thought to have lost much of its former oceans and atmosphere to space in part due to the direct impact of the solar wind." How this would play out in human timescales with respect to terraforming, I have no idea. --- Medical geneticist (talk) 22:58, 2 June 2010 (UTC)[reply]
On human timescales, it would have very little effect. The Earth effectively loses its magnetic field for a few years every few tens of thousands of years (see Geomagnetic reversal) and our atmosphere doesn't disappear. On geological timescales, it could easily have a very significant effect and may well be the leading reason for Mars not having a thick atmosphere now. --Tango (talk) 16:21, 3 June 2010 (UTC)[reply]
"This is of course ignoring the HUGE question of "where the heck to we get a planet's supply of water and oxygen from in the first place?"" -- Isn't there already a "planet's supply of water" frozen in the Martian polar ice caps? 67.170.215.166 (talk) 05:25, 4 June 2010 (UTC)[reply]
I think the supply of water is presumed to be coming from the asteroid belt and from comets, where some sort of renewed "bombardment" of the surface of Mars would happen to quickly place that water on Mars... especially while it is mostly uninhabited. A good description of how this would happen can be found in the Mars trilogy of books written by Kim Stanley Robinson. While certainly these books are works of fiction, they do provide real scientific thought on the topic including several competing methods for adding some bulk to the atmosphere. That really is the big issue, as once there is some bulk to the atmosphere you can then use plants and some bio-engineering to convert CO2 to water and other compounds needed for life. If you haven't read these books but are interested in Martian terraforming, these are simply must-read books... at least to carry on intelligent conversations with others who are discussing this concept.
Something else to consider is the Gaia hypothesis, where perhaps the introduction of living things on Mars would also sort of help to also foster an environment that would be favorable to other living things. I admit this is incredibly controversial and its application to terraforming is even more controversial still, but it is at least something to look at. It isn't entirely clear that Mars is completely without life either, although the conditions that Martian life forms are living under would be obviously different than what we find on the Earth and would be more like extremophiles if any are actually found. --Robert Horning (talk) 01:33, 6 June 2010 (UTC)[reply]
How much water Mars still has is subject to HUGE debate...but I don't think anyone would argue that there is enough to produce the picture above...especially if you are going to be using this water as a source for your oxygen (of which there is practically zero in Mars' atmosphere currently).
The "ice" caps are mostly frozen carbon dioxide, and they "melt" substantially every summer (CO2 does not melt but sublimates, or turns directly into gas).-RunningOnBrains(talk) 20:19, 6 June 2010 (UTC)[reply]
Why use the water as a source of oxygen, when we could introduce some blue-green algae into the water and use them to turn the CO2 in the atmosphere to oxygen? 67.170.215.166 (talk) 00:39, 7 June 2010 (UTC)[reply]
I believe there is oxygen contained in the molecules that make up the crust of the planet. ScienceApe (talk) 02:22, 7 June 2010 (UTC)[reply]
Well, if you use CO2 from the atmosphere, you are still left with an atmospheric pressure that is too low to comfortably support life. -RunningOnBrains(talk) 15:16, 7 June 2010 (UTC)[reply]
To clear up a possible misconception, Photosynthesis#Light reactions, including those of blue-green algae, obtain the oxygen from water and not from carbon dioxide. -84user (talk) 18:33, 7 June 2010 (UTC)[reply]

Linde's chaotic inflation: why only one level of bubble universes?

Andrei Linde's theory of bubble universes (I don't know if "bubble universe" is the acccepted term, but you know what I mean) describes them as emerging from an ur-universe (again, I don't know the accepted term). However, from what I can find on wikipedia, Linde's theory doesn't describe bubble universes emerging FROM the first "level" of bubble universes (The "level" emerging from the ur-universe); and the previous statement can be extended to bubble universes emerging from the bubble universes that emerge from the bubble universes that emerge from the ur-universe. In other words, Linde's description seems to involve only one "level" of bubble universes.

However, theoretically, bubble universes could emerge from within THIS ("our") universe, which implies at least one more "level" of bubble universes.

Have I misunderstood Linde's description? Or, perhaps, are further "levels" implied by Linde, but he leaves the implication unstated in order to simplify his description? Have others theorized further "levels"? If so, have they described this as being different from Linde's description, or just a corollary to it? 63.17.75.70 (talk) 04:04, 2 June 2010 (UTC)[reply]

In the more esoteric but serious sciences called cosmology and superstring theory, there are many theories of bubble universes, these theories sometimes overlap. For a popular science sampling, you could search for "bubble universe" at www.newscientist.com. It's one of their favourite topics, with repeated coverage. From what I've seen, most such theories imply sometimes a chain of bubble universes of unspecified length, towards both the past and the future, and sometimes that our universe bubbled forth from something "simpler" like some kind of vacuum that's not a typical universe in its own right. Sometimes the future bubbling out of our universe involves black holes, sometimes something catastrophic happening to the vacuum energy. It's all theory and you can really pick and choose. EverGreg (talk) 12:07, 2 June 2010 (UTC)[reply]
The diagram I saw (colloquium by someone... perhaps a former student of Linde?) showed sub-universes forming in a sort of bifurcation pattern, so that, if I understand your question right, this universe may already be seeding other universes, but obviously we don't see it (the bubbles form outside our dimensions). That would describe many layers of bubbles. The bifurcation diagram in chaos theory is a nice illustration, though the chaos cutoff doesn't occur I don't think. SamuelRiv (talk) 18:09, 2 June 2010 (UTC)[reply]

Thanks -- although I asked a sub-question about "corollaries" and the like, my question is about Linde, specifically. Does his theory describe only one "level" of bubble universes (each one emerging from the ur-universe, and none in turn creating new bubble universes)? That's how it seems to me, but I can't understand why he would stop at one "level." 63.17.89.8 (talk) 02:19, 3 June 2010 (UTC)[reply]

Hi, I'm hoping to clarify - the lecture I saw was directly about these bubble universes, and distinctly showed multiple levels, if I understand your meaning correctly, in that our universe may be currently seeding many more universes. Incidentally, the model is very similar to that of a bubble, in which false vaccum (a nice positive energy state that's fairly stable and forms randomly) expands until it can't hold itself up, then either bursts (kaboom) or splits (or bifurcates) into smaller universes that repeat the process, such that there are presumably innumerable levels. SamuelRiv (talk) 02:31, 3 June 2010 (UTC)[reply]
As far as I know, in Linde's picture, the large flat regions (like the one we're in) are not chaotic enough to trigger further inflation. In other words, there's only one level. Other people (like Lee Smolin) have attached additional inflationary regions to black hole interiors, making for many levels. You shouldn't take any of this stuff very seriously, though. -- BenRG (talk) 09:35, 3 June 2010 (UTC)[reply]
BenRG -- thanks, that's exactly what I was looking for (does anybody else have more?). I disagree with you about not taking it seriously. THIS universe can (possibly) create bubble universes; therefore, it's reasonable (via avoiding anthropomorphic "specialness") to presume that this universe is in turn a bubble universe. Apparently, from what you've said, Linde either does not think that this universe can create bubble universes, or else did not describe such a model. 63.17.46.82 (talk) 04:40, 4 June 2010 (UTC)[reply]
Aha, it was Lee Smolin whom I saw give the lecture. I feel like a loser for not remembering, but I was a lowly sophomore at the time, and am not in cosmology. For what it's worth, I take it seriously - it's a very "philosophically comfortable" idea for me. And look... look - bear with me here - cosmology beyond the observable universe has to have a component of philosophical comfort, right? Comfort can certainly change with our changing perception of physics, but that is as much a basis for study as any, right? SamuelRiv (talk) 05:15, 4 June 2010 (UTC)[reply]
SamuelRiv, thanks for your responses -- they were useful, but (as you see) it was Linde I was after. But this does bring up another question (if anybody's listening): Lee Smolin wrote a book advocating (or maybe just describing the theory of?) "baby universes." Years later, in a celebrated comment, he said "This universe is the only one there is." I can't figure out if this means he abandoned the baby universe theory altogether, or if he was defining "universe" to mean "the universe that contains all the baby universes." Does anybody know how Smolin's two positions relate to each other? Also, again, I would appreciate more insight into Linde's position if anyone has any.63.17.83.221 (talk) 03:20, 5 June 2010 (UTC)[reply]

Lake Okeechobee

Is there any evidence such as shocked quartz, a gravity anomaly, and tektites in areas surrounding Lake Okeechobee in Florida, USA to suggest the lake might be the sight of an impact creator as far back as the end of the Cretaceous period? 71.100.8.229 (talk) 04:48, 2 June 2010 (UTC)[reply]

Lake Okeechobee hardly even exists, in a geological sense. It is 13 feet deep at its deepest point. There is a geologist named E. J. Petuch who suggested in the 1980s that the Everglades are the remnant of an ancient crater formed by a meteorite strike 38 million years ago, but he seems to have backed away from that idea, and I can't see any serious suggestions that apply to Okeechobee. The only thing in its favor is that it is sort of roundish. Looie496 (talk) 05:34, 2 June 2010 (UTC)[reply]

Reverse jet filter - pressure drop

Hi Can anyone tell me how to calculate the pressure drop in a reverse jet filter if I know the volume flow rate and dimensions of the filter bags, no and material of bags etc Thanks —Preceding unsigned comment added by 123.237.93.244 (talk) 05:36, 2 June 2010 (UTC)[reply]

Try Bernoulli's equation —Preceding unsigned comment added by 79.76.159.3 (talk) 09:21, 2 June 2010 (UTC)[reply]

Bernoulli's equation has a term for losses. It is that term which I am interested in. So if anybody has a model or correlation for obtaining the pressure loss in the reverse jet filter I will be grateful if you can share it. Thanks —Preceding unsigned comment added by 123.237.93.244 (talk) 09:44, 2 June 2010 (UTC)[reply]

What about this ref? [1] —Preceding unsigned comment added by 79.76.159.3 (talk) 23:50, 2 June 2010 (UTC)[reply]

mortality percent

What waist to hip or other body measurement ratio has the least percent of death for each age over 60 for males? 71.100.0.241 (talk) 08:50, 2 June 2010 (UTC)[reply]

Death is affected by many other factors, so hip measurement may only be a minor one. --Chemicalinterest (talk) 11:12, 2 June 2010 (UTC)[reply]
Yes, it is an indicator of heart disease and diabetes (fatter=more likely . suprised?) specifically it's relation in general to Body mass index eg very generalised or just google for "waste hip ratio +disease", and choose a disease.
Skinnys die too - and the percentage of death is close to 100% :)
Briefly covered at Waist-hip ratio there's a link in there too. 87.102.114.166 (talk) 12:29, 2 June 2010 (UTC)[reply]
LOL... the picture caught me by surprise... the article suggests that there is a correlation between WHR and offspring intelligence due possibly to more polyunsaturated fats the fetus needs for brain development. Maybe the real reason is that the fetus just wants to find ways to attract a mate with large quarters. :-] 71.100.0.241 (talk) 15:15, 2 June 2010 (UTC)[reply]
more[2] suggests 0.95 for men and 0.80 women is a good bet, these figures are for specific causes of death , this [3] for all causes of mortality death rate increases with waste to hip ratio.87.102.114.166 (talk) 13:11, 2 June 2010 (UTC)[reply]
This has some data. [4] I can't find the exact data you ask for.87.102.114.166 (talk) 14:18, 2 June 2010 (UTC)[reply]
This http://www.nhs.uk/chq/Pages/849.aspx?CategoryID=51&SubCategoryID=165 says your waist should be under 37 inches, for men. I would doubt there is a simple optimum - you should be slim but not literally starving, but sometimes becoming thin is a sign of having a serious illness rather than dieting, and this would distort the results upwards. 92.28.249.38 (talk) 14:38, 2 June 2010 (UTC)[reply]

Oxidation of copper by iron(III) chloride

When I placed copper in a solution of iron(III) chloride, the brownish yellow solution turned green. Is it due to this oxidation reaction: Cu + 2 FeCl3 → CuCl2 + 2 FeCl2 Thank you. --Chemicalinterest (talk) 11:14, 2 June 2010 (UTC)[reply]

Think so Fe3/Fe2 standard potential is 0.77V , Cu2/Cu is 0.34V - the reaction would go, copper (II) chloride is green , ferrous chloride is pale. The potentials for chloride complexes are different, and I haven't bothered to find them since there's no big difference between copper and iron in this respect.87.102.114.166 (talk) 12:35, 2 June 2010 (UTC)[reply]
Or .. if you'd read Iron(III)_chloride#Industrial you'd have already seen the answer .. ... yes .. 87.102.114.166 (talk) 12:39, 2 June 2010 (UTC)[reply]
That's where I got my idea from; it was used as a copper etchant. It is unlike other metal-metal salt redox reactions in that another metal is not formed. I didn't see any copper(I) chloride intermediate like they stated in the article. --Chemicalinterest (talk) 13:33, 2 June 2010 (UTC)[reply]
I don't think the CuCl will be present in large amounts - it might not dissolve depending on the concentration of Cl- in solution. There might be some on the surface of the copper - eg use copper + FeCl3 dilute - remove the copper and wash the surface with (pure) water - a patina of CuCl (might) be present - any CuCl2 would be washed away by the water.
CuCl is white - not sure how to tell if it's in that solution (as CuCl2-) anyway. 87.102.114.166 (talk) 14:10, 2 June 2010 (UTC)[reply]

drug interaction checking

I perhaps did not make myself clear in a previous question regarding Phenylephrine, which is given to dilate pupils and has the side effect of restricting blood vessels and Simvastatin, which is given to reduce high Cholesterol which is the intended effect. Although there is no direct interaction between Phenylephrine and Simvastatin for a drug interaction checker to find it does not appear that any drug interaction checker will also look for the combination of blood vessel restriction from high Cholesterol and from the side effect of Phenylephrine which could be deadly. Is there a program or method which checks for these types of deadly interactions since drug interaction checkers appears to ignore such interactions? 71.100.0.241 (talk) 13:11, 2 June 2010 (UTC)[reply]

Why do you believe that they don't check for such interactions? 75.41.110.200 (talk) 15:12, 2 June 2010 (UTC)[reply]
Because I can find all sorts of drug interaction and WHR and BMI warning check sites on the Internet but not any that warn of this problem. 71.100.0.241 (talk) 16:49, 2 June 2010 (UTC)[reply]
Standard checks for phenylephrine show up various warnings and cautions for people with clogged veins etc [5] [6] [7] 87.102.114.166 (talk) 17:39, 2 June 2010 (UTC)[reply]
What makes you think that this is a problem? I mean, there could be drug-drug interactions between any combination of medicines, but why would you think that a "drug interaction checker" would look for a combination of phenylephrine and high cholesterol? It just doesn't make sense. We normally associate high cholesterol with atherosclerosis, which typically affects large blood vessels and is associated with acute blockages due to thrombosis. The mechanism of phenylephrine is completely different, as it acts primarily to constrict small blood vessels in the periphery. Can you cite any evidence to suggest that the combination of phenylephrine and "blood vessel restriction from high cholesterol" could be "deadly"? Perhaps the reason that the drug interaction checkers ignore this is that it doesn't exist? If this is something that you are truly concerned about, you need to ask your doctor about it. --- Medical geneticist (talk) 22:50, 2 June 2010 (UTC)[reply]

Remote sensing and GIS

1.Write the short notes in image enhancement techniques based on the followings;-

a)Constract stretch.
b)Image filtering.
c)Colour composing.
d)Vegitation index.

2.Write the short notes on digital image classification;-

a)Minimum distance classifier.
b)Box/parallelopipe classifier.
c)Maximum Lixelihood classifier. —Preceding unsigned comment added by Kamtwanje,Hussein George (talkcontribs) 16:01, 2 June 2010 (UTC)[reply]
Please do your own homework.
Welcome to the Wikipedia Reference Desk. Your question appears to be a homework question. I apologize if this is a misinterpretation, but it is our aim here not to do people's homework for them, but to merely aid them in doing it themselves. Letting someone else do your homework does not help you learn nearly as much as doing it yourself. Please attempt to solve the problem or answer the question yourself first. If you need help with a specific part of your homework, feel free to tell us where you are stuck and ask for help. If you need help grasping the concept of a problem, by all means let us know.--Tagishsimon (talk) 16:04, 2 June 2010 (UTC)[reply]
Remote sensing and GIS might help. Pfly (talk) 16:04, 3 June 2010 (UTC)[reply]

History of Science: What did Islamic Science invent or discover?

Does anyone on Wikipedia, have a list of scientific discoveries that can be directly attributed to scientific researchers working in the Islamic world?

Sfan00 IMG (talk) 16:20, 2 June 2010 (UTC)[reply]

We have a very extensive article on that topic, inventions in medieval Islam. Looie496 (talk) 16:24, 2 June 2010 (UTC)[reply]
Also Timeline of science and engineering in the Islamic world for other periods.87.102.114.166 (talk) 16:45, 2 June 2010 (UTC)[reply]

Thanks, Both articles seem to have an extensive set of further sources as well :) Sfan00 IMG (talk) 17:06, 2 June 2010 (UTC)[reply]


Hmm, Reading that led me to a question on a tangent... Could the Islamic World have developed a self-contained powered vehicle before the Industrial Revolution in Western Europe? Sfan00 IMG (talk) 21:37, 2 June 2010 (UTC)[reply]

Hero invented a cart that moved around by itself, the path could be programmed by winding a string round pegs on a barrel. Dmcq (talk) 21:59, 2 June 2010 (UTC)[reply]
Shame it wasn't a 'turtle'.  ;) XD Sfan00 IMG (talk) 22:32, 2 June 2010 (UTC)[reply]
I assume this was Hero of Alexandria.
It's possible but not quite there..
  • A steam engine - probably not able to make a boiler that withstood the pressures, and more importantly make pistons to the correct accuracy - but maybe they could have if they'd tried. Similar problem with internal combustion
  • Electrical - this seems more likely - they almost certainly had the battery - I assume the knew about magnets - however I don't thing they had made the connection between electricity and magnetism necessary to - make big magnets, and make motors.
I'd be fascinated to be proved wrong.
Are there any other forms of propulsion.87.102.114.166 (talk) 22:18, 2 June 2010 (UTC)[reply]
I'm not entirly sure an Arab steam engine is as implausible as you suggest, given that 'distillation' was a chemistry technique discovered in the Arab world (and would thus have needed appropriate vessels).

The articles linked also note that there were automata that ran on steam so...

I'm not saying that the self propelled vehicle has to be 'useful' .. Sfan00 IMG (talk) 22:33, 2 June 2010 (UTC) Other forms of power... - Water... - Sand ... Clockwork mechanisms - Comments on the #wikipedia-en IRC suggested mechanical clockwork power needs a coil spring which would need a certain level of metalworking. That said, simple mechanical power storage like a twisted cord could be built on fairly simple technology...[reply]

Yes spring power - in medieval time they definately had the metal working skill to make steel suitable for springs - at least Damascus steel is getting near to a similar thing.83.100.183.63 (talk) 22:53, 2 June 2010 (UTC)[reply]
The thing here is that they didn't develop such a thing. Could they (hypothetically) have done? Well, no. After a burst of creativity and invention in the 9th to 12th centuries (roughly) - things pretty much shut down. Look at the list of inventions - hardly anything in the 15th century - pretty much nothing after that. What stopped them from inventing the car (or the bicycle for that matter) was what stopped them from progressing at all since then. What stopped them was largely societal - but that's every bit as much of a brake on progress as (say) inadequate understanding of mathematics stopped the Romans and lack of interest in experimentation stopped the Greeks. In the end, the only inventions that mattered in the western world was aggressive concentration on universal scientific education and the development of "The Scientific Method". SteveBaker (talk) 23:26, 2 June 2010 (UTC)[reply]
Whoa nelly, Steve! That's a bit of overt oversimplistic Eurocentricism, don't you think? The question of Enlightenment and Industrialization is extremely complex, and in the case of the former, it's to argue that the Hellenes or Islamic scholars were not as scientifically progressive during their prime as the Royal Society at their founding, say (especially since they didn't have the advantage of a bunch of infinite series and algebraic constructs already pre-written for them). Also, universal scientific education? Not until the mid-19th century. The Scientific Method? Not until the mid-19th century. At least, not in a more advanced state than aforementioned scholars. SamuelRiv (talk) 02:19, 3 June 2010 (UTC)[reply]
Not Eurocentricism so much as counting the number of entries in Timeline of science and engineering in the Islamic world for each century. You can do that for yourself - and I'm sure you'll come to the same conclusion that I did. If you look deeper, the situation is even worse than that. So many of the recent entries are the obscure and not-earth-shattering work of a single guy who either works in the west or was trained in western universities. Several of the 21st century breakthroughs are incredibly trivial inventions (a non-glare headlamp and an automatically raising ladder for chrissakes!) - these hardly rate up there with decoding the human genome, building the Internet, writing Wikipedia, flying to the moon, and other modern inventions. I know this sounds terribly non-politically-correct, but just read what the article says...it's undeniable. SteveBaker (talk) 15:06, 3 June 2010 (UTC)[reply]
The scientific method was adopted since the Scientific Revolution. Certainly by the 19th century, the accepted way of doing science was by experiment, not by endless philosophizing or appealing to religion. --Bowlhover (talk) 07:04, 3 June 2010 (UTC)[reply]

I just noticed the 16th C Taqi_al-Din_Muhammad_ibn_Ma'ruf#The_Sublime_Methods_of_Spiritual_Machines a description of a turbine and a reciprocating engine. I still haven't found evidence that they would have been able to make suitable high pressure cylinders or pistons though.. I think a key technology here is the Lathe for metalworking but I can't find much about this aspect (they had lathes for lenses though) 83.100.183.63 (talk) 11:34, 3 June 2010 (UTC)[reply]

One of the biggest issues for the slowdown was that while a lot of the basic science was developed by the end of the 16th Century actually exploiting it to make significant further developments needed an increasingly industrial economy. That required access to suitable raw materials in significant volumes and access to significant workforce.
That didn't really start to kick in until the late 17th Century in Europe. The basic science was essential to the industrial revolution, but building on the basic science wasn't really an option in North Africa and Persia.
ALR (talk) 15:57, 3 June 2010 (UTC)[reply]
The Industrial Revolution article does list theories for why it occurred where and when it did. Most of the sociocultural arguments I believe are bunk (Christianity encouraging belief in rational laws?). The most convincing argument I have seen, in contrast to what ALR says above, is in comparative popular history, which set a standard of per-capita income. In all major advanced societies - Rome, the Ottomans, Tang China, and possibly others depending on the analysis - while productivity and scientific advancement were enormous, there were essentially too many poor people providing too much cheap labor to make machinery a worthwhile investment as it was in 1800 England.
Bowlhover-"the accepted way of doing science was by experiment, not by endless philosophizing or appealing to religion" - tell that to Ptolemy or Archimedes or Galileo or Boyle, if we only want to stay in the Western Hemisphere. Conversely, tell it to Faraday or the anti-positivist Maxwell. SamuelRiv (talk) 16:00, 3 June 2010 (UTC)[reply]
Hmmm ? Maybe I misunderstand your point, but Galileo Galilei, Robert Boyle and Michael Faraday are well known as experimentalists. Gandalf61 (talk) 08:57, 4 June 2010 (UTC)[reply]
See also Al-Jazari#Mechanisms and methods.Email4mobile (talk) 15:12, 6 June 2010 (UTC)[reply]
And Taqi_al-Din_Muhammad_ibn_Ma'ruf#Mechanical_treatises section which describes his invention of an early practical steam turbine.--Email4mobile (talk) 15:42, 6 June 2010 (UTC)[reply]
Same thing that Hero of Alexandria did in the 1st century BC. (I wonder if this might have been a case of patent infringement?) 67.170.215.166 (talk) 00:42, 7 June 2010 (UTC)[reply]
The patent would have probably expired by then. Googlemeister (talk) 14:07, 7 June 2010 (UTC)[reply]

density of liquid mixture

i would like to know how do we calculate the density of liquid mixture.for eg how do i calculate density of 40 % ethanol v/v?we know the density of absolute ethanol and that of water but how do we apply this knowledge to calculate density of the mixture?please guide. —Preceding unsigned comment added by 180.149.53.9 (talk) 19:40, 2 June 2010 (UTC)[reply]

It's not easy or impossible - you can estimate the density by averaging the densities of the components eg for 40%v/v ethanol water the density would be estimated to be 0.40 x density ethanol + 0.60 x density water.
The actual figure may be different due to the way different shaped molecules fit together as well as short range bonding interactions that differ between different types of molecule, and other factors.
In fact the estimate will be pretty close for most mixtures. Only rarely do significant deviations occur.87.102.114.166 (talk) 20:04, 2 June 2010 (UTC)[reply]
Actually ethonol is miscible with water, so mixing 1 liter of 100% ethonol with 1 liter of 100% water will not give you 2 liters of liquid. Googlemeister (talk) 20:08, 2 June 2010 (UTC)[reply]
It's separate from miscibility...the question (and solutions (sorry:) ) offered apply for any ratio that does give a solution, even if the components are not soluble at other/all concentrations (miscible). DMacks (talk) 20:12, 2 June 2010 (UTC)[reply]
The gory math for "actual" (non-"ideal") liquids is in our volume of mixing article--the intro/definition gives a nice qualitative overview (and specific example for ethanol/water) and then quickly gets to illustrating just now "not easy" the real situation is. DMacks (talk) 20:09, 2 June 2010 (UTC)[reply]
The deviation from simple "sum of parts" is noticeable enough for ethanol/water that there are official tables for converting measured density of such mixtures into their component ratio--wouldn't need that if it were directly additive because could just solve the linear combination I guess? DMacks (talk) 20:20, 2 June 2010 (UTC)[reply]
(EC with all above) For this example you would take the average of 4 parts ethanol and 6 parts water (since that is what it is composed of. The density of water is 1 g/cm3 and ethanol is 0.789 g/cm3. It would come out to (1 g/cm3·6) + (0.789 g/cm3·4) = 9.156 ÷ 10 = 0.9156 g/cm3 --Chemicalinterest (talk) 20:12, 2 June 2010 (UTC)[reply]
87's formula is much easier. --Chemicalinterest (talk) 20:14, 2 June 2010 (UTC)[reply]

plumbers putty

i have black plumbers putty sealing my bathroom sink. my sink handle was creaking so i sprayed some pam on it. the next day the pam appeared to eat away the plumbers putty and turn it brown. how is this possible? --Alexsmith44 (talk) 19:53, 2 June 2010 (UTC)[reply]

What's 'pam' did you mean PAM (cooking oil) ? If so the oil in 'pam' will have diluted/dissolved the linseed oil in the Plumber's Mait
eg as an experiment add oil (any) to putty - you'll see that it gets thinner and more runny the more you add.
Is a 'sink handle' a Tap (valve) ? 87.102.114.166 (talk) 20:08, 2 June 2010 (UTC)[reply]


yes i mean PAM (cooking oil). even if the pam dilutes the linsead oil why does it turn brown? and how does it eat away at it if the outer layer is hard? —Preceding unsigned comment added by Alexsmith44 (talkcontribs) 22:04, 2 June 2010 (UTC)[reply]

yes - putty goes hard over time - it may be evaporation, though it may also be polymerisation of the linseed oil - nevertheless hard things can be softened by things that dissolve them.
As for the brown colour - it was black before? Maybe the spray is diluting or washing away the pigment (v dark brown = black). It's difficult to say when I can't see it.
Have you got any spare putty - if so put a little in some oil and see what happens - it's possible that the colour in it will leach out (like dyes running from clothes in the wash). There still might be another explanation.
If you get stuck here you could as a last resort contact the manufacturer of the putty - they might even be interested that the colour is fading if its a flaw in their product.87.102.114.166 (talk) 22:10, 2 June 2010 (UTC)[reply]
BTW, once installed you shouldn't really be able to see the putty anymore. Its job is to sit between surfaces, not on top of them. The putty that is on top can be removed. Ariel. (talk) 00:28, 3 June 2010 (UTC)[reply]


i dont have any spare puddy i didnt install it. i used to use peanut oil to lubricate it and it didnt ruin the puddy. why did the pam they are both oils? —Preceding unsigned comment added by Alexsmith44 (talkcontribs) 00:40, 3 June 2010 (UTC)[reply]

I don't know about the particular product you used, but a number of spray oils contain lecithin as a significant component. Unlike the triglycerides in "regular" oil, lecithin acts as a surfactant/emulsifier, and as such may act differently in solubilizing/leaching compounds. -- 174.24.200.38 (talk) 03:54, 3 June 2010 (UTC)[reply]


so what should i use to lubricate it  ? it needs to be a aerosol

Trigonal Bipyramidal Molecular Geometry

Hello. How was the trigonal bipyramid derived as the shape that allows bonding electrons stay as far from each other as possible? The bond angles in tetrahedral molecules equal. Why can't trigonal bipyramids? Thanks in advance. --Mayfare (talk) 21:35, 2 June 2010 (UTC)[reply]

Note that the square antiprism is also a minimum energy configuration even though a cube would give all the angles equal. You can't arrange 5 points evenly into a regular polyhedron Dmcq (talk) 21:48, 2 June 2010 (UTC)[reply]
how was it derived.. basically by adding an extra 'arm' to the tetrahedron, or arms to the flat triangular arrangement, and then fiddling to get the least variance in distane between arms.
or from knowledge of the various three dimensional polyhedrons (most simple ones were known early in human history) - and selecting from those that had five vertices.83.100.183.63 (talk) 22:42, 2 June 2010 (UTC)[reply]
What these guys are getting at is that the shape taken is the one that minimizes the energy associated with electrostatic repulsion. For some numbers of electron pairs, like 2,3,4, and 6, the minimum energy configuration is indeed one that gives equal bond angles. In the case of 5 and 7, there are no possible ways to arrange the electrons to give equal bond angles, and instead you get a trigonal bipyramid and Pentagonal bipyramid molecular geometry, respectively. If you don't believe that there are no ways to arrange 5 or 7 bonds around a central atom that gives equal bond angles, try it yourself. As to how it was discovered that these types of molecules like Phosphorus pentachloride take this geometry, I don't know. It could have been theoretically deduced to be the lowest energy configuration (and then actually measured somehow), or the geometry may have been deduced for individual molecules, and then generalized in VSEPR theory. There's not much history that I could find. As Dmcq points out, when you get to the case of 8 bonds around an atom (very rare), there is a way for those bonds to have constant bond angles; the bonds could lie at the corner of a cube. However, that's not the least energy configuration. So it's not true, in general, that a configuration that gives constant bond angles is necessarily the configuration that a molecule actually attains. Buddy431 (talk) 22:55, 2 June 2010 (UTC)[reply]

What are we seeing when we look at the sun?

Well we all know that the sun is mostly hydrogen. But hydrogen is invisible, and when hydrogen burns, it burns invisible too I believe. So what exactly are we looking at when we look at the sun? ScienceApe (talk) 22:24, 2 June 2010 (UTC)[reply]

You're looking at the light emitted from the sun.83.100.183.63 (talk) 22:30, 2 June 2010 (UTC)[reply]
(ec) Conventional wisdom says that it emits light because it is hot - white hot in fact (in the same way a Incandescent light bulb emits light because it is hot).83.100.183.63 (talk) 22:37, 2 June 2010 (UTC)[reply]
Thermal Radiation, essentially. Anything with a temperature above absolute zero "glows". That is, it releases electromagnetic radiation at a variety of wavelengths dependent on how hot it is. Humans glow mostly in the Infrared range. The sun glows across a wide range of wavelengths, including Visible light. Buddy431 (talk) 22:36, 2 June 2010 (UTC)[reply]
I was going to suggest Blackbody radiation but that just redirects to the already linked thermal radiation article.. Vespine (talk) 23:03, 2 June 2010 (UTC)[reply]
Hydrogen burns in oxygen with a colorless flame - but what's happening in the sun is nothing to do with 'burning' - that's a confusing term! The hydrogen is participating in fusion reaction where two atoms of hydrogen are crushed together to make a helium atom and producing a heck of a lot of heat and light in the process. The color of sunlight corresponds to Black body radiation at 5800 degC - which is the surface temperature of the sun. You'll need to read the black body article to understand the precise reason for that. The color (if you were out in space) would be almost perfectly white - it only looks yellowish here on earth because so much of the blue light is scattered out there into the sky (white minus blue equals yellow!). SteveBaker (talk) 23:13, 2 June 2010 (UTC)[reply]
Strictly speaking, 4 hydrogen atoms create one helium atom, in the Proton-proton chain reaction (mostly) in the sun. Buddy431 (talk) 23:30, 2 June 2010 (UTC)[reply]
Neatpicks!!
  • The use of the word burn to describe the nuclear reaction that happens in the sun is pretty well stablished language.
  • To say that the sun "is almost perfectly white" is a pretty subjective thing to say. It looks pretty white to me even after its light's been filtered by the atmosphere.
Dauto (talk) 01:58, 3 June 2010 (UTC)[reply]
I'm still a little confused about the "color of the Sun" thing. I tend to think the main reason we think of the Sun as yellow is that that's the color when it's low on the horizon, but not quite low enough that we'd call it sunset (when it's orange-red). When it's higher in the sky, it's painful to look at and we usually don't.
But when you see the disk of the Sun through a thin layer of cloud, it appears pure white. So the question is, does the cloud layer change the perception of color, or does it just reduce the intensity enough to look at?
On the other hand, I do have the sense that sunlight on objects appears yellowish. But I can't quite figure out what that even means. --Trovatore (talk) 07:25, 3 June 2010 (UTC)[reply]
Trovatore, our perception of color is as much a consequence of our eyes and brains phisiology as it is a consequence of the spectrum of the light itself. The eyes and brain do extensive processing that allow us to have a fairly consistent color perception regardless of the king of light used to shine on the object. Because of that the question of whether the sun is white or yellowish is more or less a meaningless question. From a purely spectroscopic point of view the sun is actually greenish, but we don't see that. To answer your question, light that goes through a thick atmospheric layer becomes yellowish -> orangish -> redish while light that goes through a cloud layer doesn't, simply becoming dimmer. Dauto (talk) 15:13, 3 June 2010 (UTC)[reply]
  • Sure it's established language, but the original poster spoke of how hydrogen "burns invisible I believe", and that's about combustion with oxygen, not fusion. When the question shows signs of confusing two concepts, it's necessary to distinguish them. --Anonymous, 04:36 UTC, June 3, 2010.
Of course we should clarify any confusion or misconception that the OP might have, but we should nopt tell them not to use the word 'burn' when that's exactly the word used by the specialists in the field. Dauto (talk) 15:00, 3 June 2010 (UTC)[reply]
We're seeing photons that were created eight minutes ago. When hydrogen atoms fuse into helium, that resulting helium has slightly less mass than the hydrogen did. From E=MC-squared, you can see that the tiny loss of mass (M) turns into a relatively huge amount of energy. That energy is released as photons, an infinitesimal percentage of which reach our eyes (I may have the following wrong: I believe one-third of the photons are "visible," and the other two-thirds are invisible gamma rays). Incidentally, the CNO cycle may be a second type of solar fusion, and it also releases photons. 63.17.89.8 (talk) 02:13, 3 June 2010 (UTC).[reply]
The CNO cycle is a different set of reactions that has the same net effect as the p-p reactions - That is the conversion of hydrogen into helium. Dauto (talk) 02:46, 3 June 2010 (UTC)[reply]
The photons created by nuclear fusion in the Sun are reabsorbed. The light we see comes from the surface of the Sun and is blackbody radiation, as already mentioned. Possibly you're thinking of neutrinos, ~2/3 of which are invisible to certain detectors in a formerly mysterious way (the solar neutrino problem). -- BenRG (talk) 09:28, 3 June 2010 (UTC)[reply]

What about this though? What am I looking at in this picture? Looks like a surface of lava (I know it's not, just saying it looks like that. What is all that red and orange stuff I'm seeing? ScienceApe (talk) 00:41, 4 June 2010 (UTC)[reply]

Is it really eight minutes ago though? From Sun: "The gamma rays (high-energy photons) released in fusion reactions are absorbed in only a few millimeters of solar plasma and then re-emitted again in random direction (and at slightly lower energy)—so it takes a long time for radiation to reach the Sun's surface. Estimates of the "photon travel time" range between 10,000 and 170,000 years.[42]" How many of the photons reaching the earth travelled from the core to the photosphere and then to Earth and how many are the result of black body radiation? --Rajah (talk) 22:23, 7 June 2010 (UTC)[reply]


Same exact deal...black body radiation. Lava is at between 700 and 1200 degrees C (1000 to 1500K), and if you check the black body diagram on the left here, you'll see that that's in the red/orange range. The sun, at around 6000K is in the white region. The color at which most hot objects glow is directly related to their temperature. SteveBaker (talk) 01:32, 4 June 2010 (UTC)[reply]
That's not lava... it's a solar flare! The red and orange stuff is probably thermal radiation from hot plasma (that is, emitting visible light just like the rest of the sun does). Note that solar flares are much hotter than the surface of the sun, so can spit out a lot of radiation for their size. Also realize that electromagnetic radiation can be generated in other ways besides thermal radiation; when you accelerate matter (especially charged particles) to relativistic speeds (as occurs in solar flares), photons can be emitted then too (I don't really understand the mechanism there). This is what's believed to cause Gamma-ray bursts, for example. Buddy431 (talk) 01:45, 4 June 2010 (UTC)[reply]

But when I look at that photo, I'm not just seeing photons, I'm seeing matter aren't I? I think it's plasma if I'm not mistaken, so would it be hydrogen plasma? So I guess my question is, is hydrogen visible to the naked eye when it becomes a plasma? ScienceApe (talk) 04:11, 4 June 2010 (UTC)[reply]

Yes it is the pink colours are relatively mild plasmas generated electrically in general all plasmas Plasma (physics) emit light because of the high energy they are in.83.100.183.63 (talk) 13:50, 4 June 2010 (UTC)[reply]
Sure, you're seeing photons. That's what light is. The hot hydrogen plasma (or the lava, or even your pet cat) radiates photons over a range of frequencies (colors) with a peak frequency that depends on the temperature. At the temperature of your cat, those are in the infra-red range, in the case of the lava, it's in the red/orange range - and in the case of the sun, it's pretty much white. Those photons travel outwards in all directions and some of them arrive inside your eyeball where they are absorbed by chemicals in the rod and cone cells in your eye. Hence the perception of "color".
But that's not the whole picture - your cat doesn't look black (well, unless it's a black cat!). That's because objects also reflect light - so all of those photons coming from the sun hit your cat, some of them are absorbed and the rest reflected...and again, some of the reflected ones hit your eye. So the color of the light arriving at your eye when you look at things is a combination of the light they emit and the light they reflect (and for translucent objects, the light that passes through them).
In the case of really hot things like lava and the sun's surface, almost all of the light is emitted - so the color you see depends on the emitted light spectrum - which (for most things) follows that black body radiation curve. For much cooler objects, that radiation is largely in the infrared and you can't see it because your eyes aren't sensitive in that part of the spectrum - so the only light you see coming from your cat is the reflected light. There is also reflected light coming from the sun - but there is so very little starlight reflected from the sun that all you really see is the emitted light.
If you take a chunk of iron and gradually heat it up, it starts off at room temperature, looking slivery/grey because it's reflecting light and emitting in the far infrared where you can't see it...but as it heats up, it passes the Draper point (about 525 degC) - that emitted light starts to climb in frequency and edge into the red end of the visible spectrum. What we see is that the metal starts to glow red - it's still reflecting light but the relatively tiny amount that it reflects is now dwarfed by the amount it's emitting. As the metal gets hotter, it glows orange (now it's about 1000 degrees - as hot as lava - and roughly the same color), then yellow, then eventually white. "White hot" means "about as hot as the surface of the sun" - and you have to get your metal up to about 5000 degrees to do that. SteveBaker (talk) 14:11, 4 June 2010 (UTC)[reply]

Contents of Organic Pesticide

I just used an organic pesticide that contains the potassium salt of various fatty acids as its main ingredient. Will this actually kill/repel/do something to insect pests or did I just waste my money on some oily, salty water?24.88.87.41 (talk) 22:41, 2 June 2010 (UTC)[reply]

Maybe somebody else can give you a better answer, but the way to test for it is to spray some plants with this, and leave others of the same type alone. See how they stack up. Falconusp t c 22:49, 2 June 2010 (UTC)[reply]
There's an explanation of how they are supposed to work here [8] It looks like they affect the cell wall of things - also works on plants [9] There's plenty of hits on http://www.google.co.uk/search?q=fatty+acid+pesticide&hl=en&start=10&sa=N - so I'd think there must be something in it ??
For aphid on apple trees I think a common treatment is to spray soap on the trees - probably does work, or at least people think so [10] /83.100.183.63 (talk) 22:50, 2 June 2010 (UTC)[reply]
This could be just what I've "heard" but i always thought there was nothing wrong with organic pesticide, efficacy wise, you just need to spray more of it and more often for it to be as effective as the non organic counterpart. That's probably just what the planet rapists want us to believe. Vespine (talk) 23:01, 2 June 2010 (UTC)[reply]
Sure if you spray enough of many things you'd kill the pest. One of the ways would be by killing the plant or failing that any organisms thats eat even a tiny bit of it. Nil Einne (talk) 00:53, 3 June 2010 (UTC)[reply]
Trouble is with "nothing wrong efficacity-wise if you just spray more" is that that costs money. Fine, if like me you're growing a couple of dozen Broad bean plants in your back garden. So over the growing season you have to buy a couple or three bottles of fatty acid pesticide at £1.49 instead of one bottle of something non-organic (if as an amateur you could get it) at £1.00. No big deal. However if you have 100 acres of beans, and you're trying to make a living out of them rather than playing at farming (like me) the financial picture's rather different. Tonywalton Talk 01:05, 3 June 2010 (UTC)[reply]
There's a good amount about them on Google - just search for /fatty acid pesticide/ and similar terms. Here's one example of many. Yes, they do work OR: I have an unholy glee in spraying a fatty acid pesticide on my roses and watching the greenfly die, although you may need to use more, or apply more often, than, say an organophosphate or pyrethrin. You don't say what you're trying to kill on what crop, but personally I'd much rather get rid of aphids on my broad beans with something rather less poisonous to humans than some insecticides. Of course you could always try Enviromesh instead, if you want to avoid being called a "planet rapist". The mesh stuff also keeps pigeons off, which is a definite plus! Tonywalton Talk 00:52, 3 June 2010 (UTC)[reply]
We have an article on Insecticidal soap: "Insecticidal soaps are used against soft bodied insects and mites ... Insecticidal soaps have been about 40-50% effective against these pests." -- 174.24.200.38 (talk) 03:49, 3 June 2010 (UTC)[reply]
I probably should have qualified the sarcasm of my planet rapist comment, i don't really believe that, i think "organic" farming is far more of a scam then conventional farming. Vespine (talk) 03:51, 3 June 2010 (UTC)[reply]
Yes I thought you were being sarcastic but wasn't sure Nil Einne (talk) 05:59, 3 June 2010 (UTC)[reply]


June 3

Propagation of bloodline

From Malaria:

Endogamy along caste and ethnic lines appear to have confined these to the Tharu community. Otherwise these genes probably would have become nearly universal in South Asia and beyond because of their considerable survival value and the apparent lack of negative effects comparable to Sickle Cell Anemia.

Do we have an article about this? I'm interested.. say that 100 Chinese people marry into the United States, and no more chinese people after that are allowed in. After hundreds of generations, in the absence of any particular traits that give Chinese people a genetic advantage, will the percentage of Americans who "have Chinese blood" from the original 100 people increase to near 100% or decrease to near 0% over a long period of time? .froth. (talk) 00:14, 3 June 2010 (UTC)[reply]

Assuming random intermarriage, the average fraction of the genome that comes from the Chinese immigrants would stay roughly the same (except for random fluctuation), but it would become more and more widely dispersed. So the fraction of people who "have Chinese blood" would steadily increase, but their average proportions of "Chinese blood" would correspondingly decrease. Looie496 (talk) 01:36, 3 June 2010 (UTC)[reply]
Genetic drift might be of interest.--Lenticel (talk) 01:59, 3 June 2010 (UTC)[reply]
Specifically, the section on fixation. At such a low introduction frequency (100 foreigners into a population of 300,000,000), the alleles that are distinctly Chinese will be inevitably lost, given enough time, as a consequence of random fluctuation. Basically, this is because the frequency will fluctuate over time, and if it ever reaches 0, it will never change from that; and because of the low introduction frequency, it is highly unlikely to reach 1, from which it would also never change. Someguy1221 (talk) 05:48, 3 June 2010 (UTC)[reply]
I really can't see how a resistance to malaria wouldn't have spread throughout Asia in a few thousand years no matter how strong their rules are against marrying outside. There must be something else in it which is a disadvantage which we don't know of yet. Dmcq (talk) 15:07, 3 June 2010 (UTC)[reply]
If the female in a marriage is of a specific ethnicity, then her mt-DNA will be passed down to all her offspring, but there is as much variation in this within an ethnicity as there is between two different ethnicities. ~AH1(TCU) 23:22, 3 June 2010 (UTC)[reply]
That's just because of the meaninglessness of the concept of "ethnicity" from a biological point of view. What does any of that have to do with this? What makes you think the relevant gene(s) is mitochondrial? --Tango (talk) 23:50, 3 June 2010 (UTC)[reply]

Chemical solution for producing infrared liquid.

Hello I have a problem. I ve made the chemical solution that produces infrared light, but only lasts for 5 mins when not in a airtight container. I need to find out, how can i make it last for a few hrs in an open aircontainer. Thanks for your time, I am eagerly awaiting your answer.If you could be kind enough to e-mail it to me on (email removed) I will be gratefull to you. Thanks. Amit —Preceding unsigned comment added by 118.90.116.113 (talk) 01:15, 3 June 2010 (UTC)[reply]

I'm sorry, but the reference desk does not provide responses via email. You'll also need to provide a lot more detail about what this solution is for us to try to answer your question. Someguy1221 (talk) 05:44, 3 June 2010 (UTC)[reply]
(EC)I do not think your question is specific enough to be answered as you've stated it. I doubt there is a generic way to increase the length of an unspecified substance's IR light production in an unsealed container. Maybe if you give us specifically what your substances are and what the reaction is that produces the light, we might have a better chance. I'm not a chemist but at a guess, your reaction is oxidizing, either accelerating or otherwise interfering with the reaction, I'm not sure there is much that can be done short of inhibiting the oxygen availability by sealing the container or flooding it with an intert gas like argon or something. Then you'll of course have to worry about what argon will do to your reaction. Vespine (talk) 05:51, 3 June 2010 (UTC)[reply]
This seems kinda basic. "the chemical solution that produces infrared light" could be a bucket of hot water. Anything that's hot (or even just warm) radiates in the infra-red...you don't need some special "chemical". The thing is probably going to stop producing IR because it's cooling down more in an open container. Just stick a heating element in the bottom of a large container of water and stop it from boiling - and it'll radiate in the IR for as long as you keep the thing plugged in! If you need more IR light than that, pick something with a higher boiling point and heat it up more. SteveBaker (talk) 14:43, 3 June 2010 (UTC)[reply]
Not a chemist, but I'd also suggest that you might be looking for something that can only reflect infrared light when exposed to only red/infrared light and not normal white light, which has a very different emission curve. Remember that reflected light is an absorption-reemission phenomenon that will depend very much on the energy of light coming in. Also, as an alternative to Steve, any kind of algae or plant life is a great emitter of infrared (aerial surveys usually use infrared cameras to quickly distinguish plant life, especially useful over water). SamuelRiv (talk) 16:07, 3 June 2010 (UTC)[reply]
You have an interesting definition of "warm" - according to my calculations, anything above 10K will have a peak thermal radiation in the IR (or higher), and that's just the peak so even things colder will emit a significant amount of IR. --Tango (talk) 21:40, 3 June 2010 (UTC)[reply]
I imagine we're talking about IR emissions observable over the background, so i doubt the OP was talking about a warm bucket of water.. I could be wrong, but that's the way i interpreted it. Vespine (talk) 03:54, 4 June 2010 (UTC)[reply]
Actually, I'd love to see what the OP had put together as his solution, and maybe I can play around with some quantum mechanics calculations to see if I can get some idea (though it has a 99% chance of being completely fruitless). Beyond that I'm completely lacking in chemistry knowledge, though I know there's plenty of literature on UV-peak solutions - maybe you can do it all backwards :). SamuelRiv (talk) 05:06, 4 June 2010 (UTC)[reply]

Cooking oil contamination?

I have a large frying pan and cook raw chicken thighs/legs for about 45/50 minutes on a low/medium setting-the oil bubbles and seems to reach high temperature for long periods. But I change the oil once a week. Bearing in mind carbonised residue build up at the bottom of the pan, could bacteria build up even in boiling oil, and how often should it be changed? The food is for my own consumption, not anyone else. —Preceding unsigned comment added by 80.1.80.13 (talk) 07:27, 3 June 2010 (UTC)[reply]

According to our cooking oil article, you should be more concerned about degradation of the oil due to exposure to air and light. Bacteria could develop in the oil when stored in the pan, and our article especially mention the risk of botulism. However, the botulism article also states that a long cooking period breaks down the botulism toxin. For more information it would probably be a good idea to look up guidelines and routines specific to restaurant and take-away kitchens.
As a side note, bacteria and fungi in general are killed by boiling, but the poisons they leave behind could survive. I seem to recall this is a problem with some yeast/fungi. EverGreg (talk) 09:33, 3 June 2010 (UTC)[reply]
"45/50 minutes" - you like your chicken well done then. What sort of oil is it, because different oils boil at different temperatures. 86.4.183.90 (talk) 18:00, 3 June 2010 (UTC)[reply]
Cooking oils don't boil at all, they break down first. There is no chance of bacterial buildup in oil that is used daily or almost-daily to fry chicken. The main reason you will want to change the oil is because stuff will leak out of the chicken and start to break down, causing it to taste gradually more nasty. Your oil will also contain steadily larger fractions of chicken fat, which is exuded by thighs and legs in large quantities when you cook them that way. Looie496 (talk) 23:07, 3 June 2010 (UTC)[reply]
...And let me add that I can hardly imagine a more unhealthy diet than eating chicken cooked that way multiple times per week. Looie496 (talk) 00:57, 4 June 2010 (UTC)[reply]
Come now, you can't being trying very hard. A diet made up of 50 Mars bars a day would surely be worse, especially if some of them were deep-fried. Or nothing but rabbits. Or, indeed, nothing but carrots. There are many more unhealthy diets that I can easily imagine, and we don't know why the OP feels the need to eat the way they do. 86.164.69.239 (talk) 20:05, 4 June 2010 (UTC)[reply]
Don't quibble - the diet indicated by the OP is still likely to shorten their life by decades, compared to an optimum healthy diet. In addition to the fat and saturated fat, high-temperature cooking creates nasty unhealthy chemicals that I do not know the details of to describe. Re-using frying oil or fat is I believe particularly bad for creating carcinogens. As I never fry food, I do not know enough to describe the details. 92.15.3.53 (talk) 10:16, 5 June 2010 (UTC)[reply]
Don't quibble? What else are we here for? :) You don't know anything about them, or what an optimum diet for them is. Certainly, most people would be well advised to have more variety in their diet, and less fat. And there is a danger in creating carcinogens with high-temperature cooking. But, again, we know nothing about this person and why they are doing what they are doing. Just for example, when my younger sibling's friend with cystic fibrosis stayed with us, they were on a high calorie, high fat diet. If they didn't eat enough fat and sugar in the form of things like mars bars, they had to eat a disgusting high-calorie paste that they hated. I made a load of buttercream which helped avoid the paste. For me, that much buttercream would be a bad diet. For them, it was a good option. So no - we do not know why the OP is eating what they are eating. We do not know what else they are eating. And there are indisputably worse diets they could be eating. 86.164.69.239 (talk) 12:40, 5 June 2010 (UTC)[reply]
There's a restaurant whose fryer hasn't been completely-emptied/filled-with-new-oil since 1912.[11] DMacks (talk) 20:21, 4 June 2010 (UTC)[reply]
I read somewhere, years ago, that when cooking oil is repeatedly used it reacts with the steam given off to create a byproduct that causes less of the oil to be absorbed, so that cooking in used oil may actually be healthier than cooking with fresh oil. In any case I find that repeatedly used oil develops an interesting flavor that can't otherwise be duplicated. When I lived in Tucson I would sometimes buy Indian fry bread. The most flavorful was always made by dumpy Navajo women with names like Nellie Begay, and their cooking oil always looked like it had been used for decades. Fry bread made with fresh clean oil tastes disappointing in comparison. Looie496 (talk) 16:57, 5 June 2010 (UTC)[reply]
I'm very skeptical about that. The discovery of the carcinogenic danger of re-using frying oil was done with a proper scientific study, although I do not recall enough to cite it. 92.24.182.48 (talk) 20:08, 5 June 2010 (UTC)[reply]
Looks to me like it would be fairly straightforward to find in Web of Science or something, if you have access at the moment (I don't).
Columbia University says, "Reusing cooking oil has been done for ages. There really isn't a problem, if done properly" and that the carcinogen danger comes from rancid oil, which is oxidised oil. Heating the oil to the smoke point will increase oxidation, so using oil with a high smoke point and keeping below that will probably help. When the oil smells and looks rancid, don't use it.
This paper from 1973 is probably interesting, if anyone can access the whole thing to skim it. Seems to be about frying chicken in new and reused oil.
This one from 1984 claims to find no mutagenic effects.
There are some other interesting looking ones I can't access at the moment, although many seem to focus on the dangers of inhaling fumes. Looks like a fun project for someone with access and some free time. 86.164.69.239 (talk) 23:16, 5 June 2010 (UTC)[reply]

Water boiling question

Firstly the actual question: "At what temperature does water boil, and how could this be confirmed experimentally?" (I'm posting this in response to some disscussions elsewhere on wiki :) )

A simplistic answer to the question might be: " Water boils at 100 degrees C. To confirm this heat some water in a kettle and measure the temperature of the steam produced."

However, there are a number of flaws and pitfalls within that answer, I can think of 3 already. :) Sfan00 IMG (talk) 14:04, 3 June 2010 (UTC)[reply]

I'm not a chemist, but if I remember my high school chemistry this assumes standard pressure. You would need a phase diagram to correctly answer this question. --Shirik (Questions or Comments?) 14:09, 3 June 2010 (UTC)[reply]
Don't you mean high school physics? Dauto (talk) 14:42, 3 June 2010 (UTC)[reply]
Well, in the UK at least, this is covered in secondary school chemistry. Vimescarrot (talk) 18:22, 3 June 2010 (UTC)[reply]
I suspect that this is in response to a previous question asked by this OP on the Ref Desk Wikipedia_talk:Reference_desk#Reference_Desk_Scope about how original research is handled at Wikiversity. The OP had suggested that doing an experiment to find the boiling point of water was a simple matter. I pointed out that there are a bunch of experimental mistakes you could make - even for such a simple question - that would mess up the answer. Hence if someone claimed to have done the experiment and gotten some value other than 100C, then I'd ask:
  1. Did you measure the temperature when the bubbles just started to appear - because gasses come out of solution before boiling point.
  2. Were you doing this at sea level or at an elevation where the air pressure is lower?
  3. Did you put the thermometer into the water as it was heating up - or just dunk it in quickly (thereby lowering the temperature of the water immediately around the thermometer)?
  4. How long did you wait for the temperature to settle down?
  5. Did you remove the thermometer from the water in order to read it? (Hint: Rookie mistake - seen it a million times!)
  6. How pure was the water? Even if it was distilled water, could contaminants have gotten into it from the container or the thermometer?
I'm not sure what else is being asked here - but I believe that's the background to this question. SteveBaker (talk) 14:50, 3 June 2010 (UTC)[reply]
More questions for completeness's sake:
  1. How did you heat the water? If it was a pot of water heated on a stove, there could be a significant temperature difference between the bottom and top of the water.
  2. If you are measuring the steam and not the water, is there any condensation on the thermometer? The latent heat of condensing water could lower the temperature registering on the thermometer.
-RunningOnBrains(talk) 20:45, 3 June 2010 (UTC)[reply]

Thanks Steve :) , This was indeed a question not only about the actual physics (for which I have a basic idea) but on scientific technique (and the pitfalls thereof)

Of the pitfalls I can think of, the one's Steve mentions do exist :), I was also considering...

  • Given the phase diagram, it's maybe possible for 'vapour' to form before the water actually boils?

Meaning that the temperature read might be of the vapour cloud, not the steam...

  • Although the simple answer states to measure the steam, it isn't more specfic, This is a consideration because (conjecture)

given how a typical 'cloud' of steam from a kettle disperses, you might get different readings depending on where in the cloud you measured, owing to variations in pressure?

  • How is 'water' defined? Steve makes a valid point about impurities. A releated consideration if not using distilled water

would be the potential for substances in solution(?) or dissolved gas, (water being a solvent IIRC)

  • The simplistic answer assumes a single measurement with a thermometer., but depending on the type of thermometer there

may be characteristics of the thermometor which affect the result obtained.

All considerations.

I suppose revised questions should be

' How do you determine the temperature at which pure water boils given identical inital conditions, in a repeatable manner?' ' What external factors would potentialy have an influence on the boiling point of 'water' ?'

which are slightly different question to those asked initially. Sfan00 IMG (talk) 15:17, 3 June 2010 (UTC)[reply]

And while we're being uber-picky, let's toss in:
  • Did your ultra-pure water have the correct ratios, deuterium, tritium, O17 and O18 in it?
SteveBaker (talk) 13:47, 4 June 2010 (UTC)[reply]
It all depends on what level of precision you want. Just sticking a thermometer into boiling water will give you the correct answer to within a few degrees every time, I can assure you. If you want to get down to a few hundredths or thousandths of a degree you need to start eliminating unknowns: Heat everything equally (atmosphere and thermometer too!), use ultra-pure water (reverse osmosis works great supposedly), and use other methods to control your environment as much as you can. -RunningOnBrains(talk) 20:45, 3 June 2010 (UTC)[reply]
Water is rather a very difficult one to measure, partly due to the very high latent heat, which means that one has to apply a high heat source to get a good boiling mixture - thus the reading in the water can be a bit higher than you expect - just try making jam, it will vary where you locate the thermometer in the pot. Another point is that thermometers are calibrated for a fixed dip - may be 76mm, may be total immersion - you need to get that correct. The ideal way is to measure a mixture of steam and boiling water far away from its initial heat source - and the only bit of kit I have ever used that works well is a "Gillespie Still" - the boiling water/steam mixture is carried up away from the heater to the thermometer. The accuracy is very good.  Ronhjones  (Talk) 20:33, 3 June 2010 (UTC)[reply]
Could you clarify what you mean by a Gillespie Still as they doesn't appear to be an article yet?

( although as an item of lab equipment it shows up in a number of scientfic links when looked for on Google..) Sfan00 IMG (talk) 22:16, 3 June 2010 (UTC)[reply]

IMHO the answer you're providing begs the question, 100 degrees Celsius is an arbitrary constant BY DEFINITION the boiling point of water. It is more correct to say that you get 100 C at the point when water boils, as opposed to water boils when you get 100 C. IMHO, to prove 100 C is actually the correct boiling point you have to NOT take it for granted, a more meaningful answer would be to prove that water boils at 387.15 K. Vespine (talk) 03:48, 4 June 2010 (UTC)[reply]
That's a very misleading statement - and our OP should ignore it. The tie between definition of a degree and the boiling point of water only holds under very specific circumstances. If you measure the boiling point of water at the top of Mount Everest, it'll be around 69 degC. We don't change the temperature represented by a degree centigrade when you're standing on the top of a mountain...so your assertion that the answer begs the question is quite untrue. You can meaningfully measure the boiling point of water and end up with an answer that isn't 100C...so "BY DEFINITION" doesn't hold here. It is also untrue that the definition of 100C is simply that of the boiling point of pure water at standard air pressure. That definition was changed in 1954 and again in 1967. These days, it's defined by the triple point of Vienna Standard Mean Ocean Water which is not just pure water, it has to have precisely the right mix of hydrogen and oxygen isotopes - so even ultra-purified water might have too much or too little deuterium or tritium in it or the wrong amounts of O17 and O18 to get precisely that 100C result. So, again, this fixed relationship concept is bogus. SteveBaker (talk) 13:47, 4 June 2010 (UTC)[reply]
Water doesn't boil at 387.15K (at least not at standard pressure) according to the relevant article..
Follow on question : ' How to define the boiling point of water in manner that allows it to be used as

a reliable calibration point?' Sfan00 IMG (talk) 11:41, 4 June 2010 (UTC)[reply]

Vienna Standard Mean Ocean Water has that explanation. SteveBaker (talk) 13:47, 4 June 2010 (UTC)[reply]
Yes you need a standard composition for the water, and you also need to measure the bp under specific conditions - the atmospheric pressure is one of the most significant factors affecting the boiling point of water. pressure/bp data here. For reliable accuracy you need to know how the bp will vary with impurities as well - so you know whether the error in experimental variables will affect the bp measurement significantly.87.102.32.39 (talk) 15:41, 4 June 2010 (UTC)[reply]
I'm considering bookmarking this thread, clear yet interesting discussions. :) Sfan00 IMG (talk) 16:22, 4 June 2010 (UTC)[reply]
Another thing you can do is to define (and/or measure) the bioling point of water as a function of different variables (such as pressure) - then when you measure the boiling point another day you can look up on a table (such as given above) - to find out what the temp. should be under those conditions - ie use a barometer as well.
if you bookmark this thread the URL will change in 2-5 days when it is archived. the actual URL will probably be http://en.wikipedia.org/wiki/Wikipedia:Reference_desk/Archives/Science/2010_June_3#Water_boiling_question 87.102.32.39 (talk) 16:29, 4 June 2010 (UTC)[reply]

Weller soldering

Good Morning, My question refers to Soldering and Weller. I've learned that my relative is Mr.Harry A.Ungar who invented the Ungar soldering pencil, Ungar soldering system, Soldering toys etc.. This has become a genealogical search as well. It seems Harry Ungar left Hungary, either from the Slovakia or Austrian areas. I've learned Mr Ungar had a plant located at 1401 Redwood, California, There is little to no biographical information that I've found but dearly need to track him back to my Great grandfather and to find living relatives.

Mr.Ungar it seems had a government contract to make soldering pencils for WW2.
Merged with Karl Weller.
Ungar/Weller sold to Cooper.
Listed in the southern Ca business directory listings1958,1961
Sidney Ungar Donated to save the Hollywood Bowl.LA Times Aug 21, 1951
His Children may be : Sidney, Leon, and Ruth Ungar(Marks).
Toys Manufacturer Assoc LA Times Dec 21, 1950 pg 25 Sidney Ungar.

I beg you! with your wonderful resources, can you help to find biographical information from any sources /media. The genealogists have accepted my hundreds of dollars but claim they are stuck. Even the above information is my work not theirs. I really need to give my children some attention and stop wasting their time as I search W/O result. Thank you in advance. Perry Urany <email redacted> —Preceding unsigned comment added by Perrykesan (talkcontribs) 16:03, 3 June 2010 (UTC)[reply]

  • I've reformatted your question slightly to avoid an ugly layout of boxes and removed your email address so you don't get spammed to death. Any answers will appear here - we don't email answers out. Exxolon (talk) 16:32, 3 June 2010 (UTC)[reply]
    • Both Ungar and Weller are very well respected companies in the soldering business, and I have been a proud owner of their products. At Google Book search the earliest ref I found to Harry A. Ungar was Boys Life, Nov 1936 page 40, where he advertised a "woodburning" kit using a little heated stylus much like a soldering pencil. (I remember getting a nice third degree burn from one at summer camp crafts). The soldering pencil as such shows up in Popular Mechanics, December 1944 page 144, for a 17 watt soldering pencil which reached full heat in 90 seconds. On page 6, his company was listed as the maker. An ad showing some models of the pencils was in Billboard Nov 24, 1945 page 96. At Google News Archive there were several stories about persons named "Harry Ungar" (including a manager of the Stewart Products Service Station in California) back to 1919 but no idea if they are the same one. The first certain one is a pay per view from the LA Times, Feb 1, 1953 about a new factory in Venice CA, where it said "Sidney D. Ungar" was company president. Edison (talk) 04:00, 4 June 2010 (UTC)[reply]

cold fusion

OK, suppose somebody invents cold fusion in order to make electricity. On an industrial scale, they take water, hydrolyse it to its constituents hydrogen and oxygen, and then fuse the hydrogen to form helium. In doing so, they save the world from global environment change. However, might this cause other separate problems? Wouldn't water run out eventually? If tonnes of helium and oxygen are released into the earth's atmosphere, wouldn't they become pollutants? Bobble hobble dobble (talk) 17:36, 3 June 2010 (UTC)[reply]

According to World energy resources and consumption: "In 2008, total worldwide energy consumption was 474 exajoules (474×1018 J)." If we use the proton-proton chain reaction as an example (there are lots of different fusion reactions - this is the one that dominates in the Sun), then we get 26.73 MeV for every 4 hydrogen nuclei being turned into helium (and some positrons and neutrinos and things, that we can ignore). Google says we need about 4x1032 hydrogen nuclei a year. That corresponds to about 6.6 million litres of water. The Earth has about 1,360,000,000 km3 of water. That's 1.36*1021 litres. So even with a massive increase in energy consumption, we still wouldn't use even a noticeable fraction of the water in the billions years or so the Earth will be habitable for. The 2x10^32 atoms of oxygen that would be released a year compares with about 4x10^43 atoms of oxygen already there, so again it wouldn't be significant over the remaining habitable lifetime of the Earth. The 10^32 atoms of helium released corresponds to 4x1039 atoms already there. So that would be significant over timescales of millions of years, but wouldn't be a problem since helium is a noble gas, so is very inert. Basically, the amount of energy released by the creation of each atom of helium is so great that you don't need to make many to meet our energy needs. --Tango (talk) 18:19, 3 June 2010 (UTC)[reply]
If there was a dirt-cheap form of energy, the first side-effects would be the collapse of the current energy infrastructure and those nations dependent on them, like Saudi Arabia. In the long term, cheap energy might lead to "bad behavior", such as heating outdoor areas, like beaches, with radiant heaters. We might eventually get to a point where global warming created by actual heaters and A/C would surpass that due to greenhouse gasses. Also note that while total water consumption isn't an issue, water levels in lakes near large populations might eventually decrease by a detectable level. StuRat (talk) 18:57, 3 June 2010 (UTC)[reply]
Simple answers: No, no, and no. Dauto (talk) 20:27, 3 June 2010 (UTC)[reply]
Release of molecular hydrogen into the atmosphere on a large scale could severely deplete the ozone layer [12]. It would depend on how much hydrogen you would need to create, and how leak-proof your systems are.-RunningOnBrains(talk) 20:55, 3 June 2010 (UTC)[reply]
You could always collect the wast H2 for portable fuel needs. Googlemeister (talk) 21:04, 3 June 2010 (UTC)[reply]
The point is to transmute the hydrogen into helium. I don't think helium would have much effect on the atmosphere, being inert. APL (talk) 21:27, 3 June 2010 (UTC)[reply]
...and very light. Atmospheric helium evaporates into outer space pretty readily, is the reason the concentration is so low even though "so much" is being released terrestrially. DMacks (talk) 21:31, 3 June 2010 (UTC)[reply]
You can burn off any leaking hydrogen and turn it back into harmless water. --Tango (talk) 21:29, 3 June 2010 (UTC)[reply]
I remember hearing about an electronic device in Germany that uses water molecules to produce electricity, but the voltage is on the order of <1.0V. ~AH1(TCU) 23:18, 3 June 2010 (UTC)[reply]
I believe the water just serves as a catalyst. The energy comes from something else in the fuel cell. Water is a very low energy state, so it is extremely difficult to get energy out of it (fusing the hydrogen is the only method I know, and the device you remember certainly doesn't do that). Of course, the media when the device was announced said it used water as a fuel, but the media were talking nonsense. --Tango (talk) 23:54, 3 June 2010 (UTC)[reply]
Would my voice start to sound squeaky with all that helium? Edison (talk) 03:58, 4 June 2010 (UTC)[reply]
No - the amount produced is utterly negligable. Of course hydrogen would also make your voice sound squeaky - but I strongly recommend that you DO NOT try that! SteveBaker (talk) 13:11, 4 June 2010 (UTC)[reply]
I've heard that the Alaska Brewing Co. actually makes beer that's "carbonated" with hydrogen instead of your least favorite gas CO2 (well, used to make it, at any rate) -- there even is (or used to be) a karaoke contest where the contestants (all men) would drink this beer and then sing high soprano songs while blowing fireballs from their mouths (using a cigarette or other ignition device to ignite the gas). I don't know if they still do any of this, though. 67.170.215.166 (talk) 01:36, 6 June 2010 (UTC)[reply]

variations of h2o

It is my understanding that H2O can only be in three states: liquid (water), gas (steam) and solid (ice) and that regardless of the state the chemical configuration is the same. So, are there other states of H2O and does the formulation remain independent of the state? 69.77.185.91 (talk) 18:26, 3 June 2010 (UTC)[reply]

The chemical formula has to stay the same, otherwise it wouldn't be water any more. There are 15 known phases of ice (Ice#Phases). It can also exist as a supercritical fluid, which is sort of a another phase of matter and has properties in common with both liquids and gasses. --Tango (talk) 19:20, 3 June 2010 (UTC)[reply]
Is it possible to create water plasma? Googlemeister (talk) 19:40, 3 June 2010 (UTC)[reply]

In a plasma the atoms are ionised and do not form chemical bonds, so it wouldn't be water anymore. A supercritical fluid isn't technically a different phase since it can be reached without going through a phase transition. Dauto (talk) 20:26, 3 June 2010 (UTC)[reply]

That's why I said "sort of" - I didn't want to get into details. I guess the closest thing to a water plasma would be a mixture of oxygen and hydroxyl ions. --Tango (talk) 20:47, 3 June 2010 (UTC)[reply]
Water is a molecule with strange properties, for example one test showed water to have a chemical formula of H1.5O even though it is not possible. ~AH1(TCU) 23:17, 3 June 2010 (UTC)[reply]
I'd love to read about that test - do you have a link? Water definitely does some odd things, but I can't think of any interpretation by which it would have a fractional chemical formula like that. --Tango (talk) 23:55, 3 June 2010 (UTC)[reply]
I don't see it mentioned in Water (molecule), but see [13] for example. DMacks (talk) 00:09, 4 June 2010 (UTC)[reply]
That article says it isn't a property of water, but rather just the way hydrogen works - they tried the same experiment with benzene and got very similar results. --Tango (talk) 00:31, 4 June 2010 (UTC)[reply]
Perhaps you're thinking of the self-ionization of water (though I think that water is not the only thing that does that)? Paul (Stansifer) 00:50, 4 June 2010 (UTC)[reply]
It's not; the article talks about properties of certain molecules containing hydrogen where, under certain circumstances, on very short time scales, some of the hydrogens seem to not exist when neutrons are bounced off of them. It appears to be some sort of quantum effect. Buddy431 (talk) 01:20, 4 June 2010 (UTC)[reply]
With self-ionisation, the ratio of hydrogen atoms to oxygen atoms stays at 2:1, they just aren't joined together in the same way. This is something quite different and it seems nobody really knows why it happens. --Tango (talk) 01:29, 4 June 2010 (UTC)[reply]
There are alternate forms of solid ice, such as ice IV. Also under extreme pressure you may form a metallic phase. Graeme Bartlett (talk) 05:36, 4 June 2010 (UTC)[reply]
Other variations with different formulas are: Hydroxyl radical, Hydroxide, Hydrated hydroxide ion H3O2, Hydronium and on a side track Dioxygenyl and peroxide. Also you can get Supercooled waterand Superheated water. Graeme Bartlett (talk) 05:57, 4 June 2010 (UTC)[reply]

interactions of colloidal gold

How strong is a bond between the surface of a gold nanoparticle and say, a lipid compared to most lipid-lipid interactions? What about polar molecules and aromatics? John Riemann Soong (talk) 18:27, 3 June 2010 (UTC)[reply]

Colloidal gold particles are charged, so they might have a small effect on polar molecules, probably not much else on any other molecules. --Chemicalinterest (talk) 20:34, 3 June 2010 (UTC)[reply]
see also Chrysiasis. Rmhermen (talk) 20:38, 3 June 2010 (UTC)[reply]
They're charged? I'm pretty sure the gold nanoparticles I am using are Au(0)... would some atoms be charged and not the others? John Riemann Soong (talk) 17:10, 4 June 2010 (UTC)[reply]
See Colloids#Interaction between colloid particles. Since they are often charged...--Chemicalinterest (talk) 23:10, 4 June 2010 (UTC)[reply]
An odd one - gold colloids are indeed Au(0) - the charge is often from citrate (from sodium citrate) stuck to the surface of the gold [14] or Cl- [15] - but they carry a negative charge - you can demonstrate this by using an electric field, see also 87.102.32.39 (talk) 00:38, 5 June 2010 (UTC)[reply]
Why do negative anions like to bind to Au(0)? Is it because of the increased surface energy of nanoparticular gold, making colloidal gold slightly electrophilic? (In a manner similar to boron?) John Riemann Soong (talk) 14:43, 7 June 2010 (UTC)[reply]

What is the reduction potential of selenium? (Se + 2 e- ←→ Se2-) --Chemicalinterest (talk) 20:36, 3 June 2010 (UTC)[reply]

-0.67V, if this tattered piece of paper in my disused chemistry notes folder is to be relied upon. ~ mazca talk 21:10, 3 June 2010 (UTC)[reply]
Sounds right. Oxide is +1.23, sulfide is +0.14, selenide is -0.67, telluride is -1.14... Thank you. --Chemicalinterest (talk) 21:18, 3 June 2010 (UTC)[reply]
Mmmh [16] gives a totally different figure - wonder why. The Table_of_standard_electrode_potentials gives a figure for Se <> H2Se - which is what would happen in standard conditions - it should be possible to work backwards from that if you've got figures for the acidity constant of H2Se87.102.32.39 (talk) 15:16, 4 June 2010 (UTC)[reply]
It could be that too, it still fits in with the periodicity of the chalcogens. --Chemicalinterest (talk) 23:07, 4 June 2010 (UTC)[reply]
That's why I didn't include any figures in the selenide article, just said they're moderately reducing. --Chemicalinterest (talk) 23:08, 4 June 2010 (UTC)[reply]

cell phone

how come my cell phone gets very hot if i talk for awile ? —Preceding unsigned comment added by Alexsmith44 (talkcontribs) 22:06, 3 June 2010 (UTC)[reply]

Its microprocessor is doing more work, e.g. digitising your voice, converting incoming sound to analogue, and ancillary electronics are also doing more - driving the speaker and the aerial system. And the battery is doing more work to provide the current for all of this. When you're not talking, very much less is happening in all departments and so it keeps its cool. (What I'm not sure about, and perhaps someone else will be, is whether there's any significant microwave -> heat thing going on with it.) --Tagishsimon (talk) 22:13, 3 June 2010 (UTC)[reply]


it feels like it the battery that gets hot —Preceding unsigned comment added by Alexsmith44 (talkcontribs) 22:26, 3 June 2010 (UTC)[reply]

It is the battery that get's hot. Your phone is using more energy. (Mostly through its radio transmitter, not its CPU) The more electricity your battery has to put out, the hotter it gets. APL (talk) 22:52, 3 June 2010 (UTC)[reply]

June 4

How does a battery-less moisture-meter work?

How would a moisture meter such as this work, bearing in mind that it uses no batteries? When inserting into soil, the needle flicks to the right before slowly moving a distance to the left and settling. Which of the readings can be considered the proper moisture level of the soil, based on how the device supposedly works? Would it be likely to harm the meter, to leave it sitting in soil, rather than removing and cleaning it every time? --92.25.237.15 (talk) 01:18, 4 June 2010 (UTC)[reply]

The picture clearly shows the two electrodes are made of different metals, so I would imagine it works in a similar way to a lemon battery. In the normal use of the device, the electrodes will be eroded (that's how it works - see the article I linked to), so leaving it in the soil will drastically reduce its working lifetime. --Tango (talk) 01:34, 4 June 2010 (UTC)[reply]
Have you tried covering the light sensor of the light meter? If the device stops working it might mean it uses the light as a source of energy. Either way, leaving it in the soil might not be a good idea. Have you tried asking the manufacturer? Dauto (talk) 02:45, 4 June 2010 (UTC)[reply]
That's a quote from the link provided by the OP "To avoid damaging the electrode, please clean the electrode after each use". I guess that answers the question. Dauto (talk) 02:50, 4 June 2010 (UTC)[reply]

Does Synthetic Vanillin / Vanilla contain Alcohol

Does synthetic Vanillin or synthetic Vanilla contain alcohol? Or is alcohol used in the process of making synthetic vanillin/vanilla? If yes which alcohols are used?---- —Preceding unsigned comment added by 212.22.185.11 (talk) 06:30, 4 June 2010 (UTC)[reply]

I think that vanillin is often dissolved in ethanol - pure vanillin would not contain any alcohol but if you were to buy a bottle of vanilla extract then it will contain 35% alcohol. 86.7.19.159 (talk) 12:21, 4 June 2010 (UTC)[reply]
I have a bottle of synthetic vanilla which contains: water, caramel color (to make it dark like real vanilla), flavors, potassium sorbate. No alcohol. --Chemicalinterest (talk) 12:42, 4 June 2010 (UTC)[reply]
Guys, vanillin is not water-soluble. It has to be dissolved in some sort of (edible) organic solvent or be stabilised with surfactants. John Riemann Soong (talk) 17:27, 4 June 2010 (UTC)[reply]
The "alcohol-free" vanilla flavorings I've seen use glycerol or other "not ethanol but similar-structure" solvent. DMacks (talk) 17:41, 4 June 2010 (UTC)[reply]
Vanillin is water soluble. Not highly soluble, admittedly, but I've regularly made 100mM solutions in water without it crashing out. Rockpocket 18:04, 4 June 2010 (UTC)[reply]
"Solubility in water 1 g/100 ml (25°C)" if we're to believe our own article. DMacks (talk) 18:10, 4 June 2010 (UTC)[reply]

JRS: "Guys, vanillin is not water-soluble. It has to be dissolved in some sort of (edible) organic solvent." Apparently, it's slightly soluble in water.

Page 3-292 of the CRC Handbook contains the following data on vanillin:

No. Name Synonym Mol. Form. CAS RN Mol. Wt. Physical Form mp/°C bp/°C den/g cm−3 nD Solubility
6060 4-Hydroxy-3-methoxybenzaldehyde Vanillin C8H8O3 121-33-5 152.148 tetr (w, lig) 81.5 285 1.056 sl H2O; vs EtOH, eth, ace; s bz, lig

The solubility abbreviations are as follows:

  • i = insoluble
  • sl = slightly soluble
  • s = soluble
  • vs = very soluble
  • msc = miscible
  • dec = decomposes

Other abbreviations:

I have three different bottles of vanilla flavouring in front of me:

Brand Product Ingredients
Dr. Oetker Vanilla – Natural Extract Water, Alcohol, Extract of vanilla
Silver Spoon Vanilla – Natural Extract Water, Ethanol, Extract of vanilla
Silver Spoon Vanilla Flavouring Water, Ethanol, Flavouring

Ben (talk) 14:12, 5 June 2010 (UTC)[reply]

Mercury and Dental Fillings

Let me preface this question with two things: this is not a request for medical advice, and I do not put any stock in amalgam fillings causing mercury poisoning. However, my question is related to the second point; supposing you took a normal sized amalgam filling and ate it, what would the effects be? What if you inhaled the equivalent ammount of mercury present? Finally, instead of ingesting/inhaling it all at once, what if you stretched out the time span to taking 1/500th of the filling every day, what would be the effects after 500 days (any number can be put in place of 500, I'm really just curious about how dangerous amaglam fillings would be, were the people opposing them correct.) 66.202.66.78 (talk) 10:43, 4 June 2010 (UTC)[reply]

Have you looked at Dental amalgam controversy which while not addressing your hypotheticals would seem the obvious starting point. Nil Einne (talk) 11:12, 4 June 2010 (UTC)[reply]
Mercury poisoning also has a lot of useful information. Between the fact that the mercury is in an amalgam with silver and is thus carried out of the body fairly easily - and the fact that the uptake of mercury through the digestive system is so low (0.01%), this is a really small problem. Mercury is exceedingly dangerous when it's reacted with other stuff - but the pure metal is much less nasty. The amount of mercury you absorb from a tunafish sandwich is likely to be greater than from accidentally swallowing a tooth filling. SteveBaker (talk) 13:06, 4 June 2010 (UTC)[reply]
Mercury is a rather inert metal, so its rate of absorption is low. Mercury compounds are reactive and toxic, and some of them are easily absorbed, making them extremely toxic. Improperly made dental fillings may be at more risk of mercury poisoning. --Chemicalinterest (talk) 14:38, 4 June 2010 (UTC)[reply]
I had a filling like this when I was a kid. I remember nausea and a lot of vomiting. --Ouro (blah blah) 15:56, 4 June 2010 (UTC)\[reply]
That may be better explained by stress or side effects of the anaesthetic, Ouro. The studies I've read show zero history of actual mercury poisoning effects in post-1950s dental fillings. SamuelRiv (talk) 17:33, 4 June 2010 (UTC)[reply]
As I recollect, I got this filling in communist Poland. I surmise that the quality might not have been that high. Or I was young, or stressed. No way to check it out. Just volunteering information, SamuelRiv. But thanks though. --Ouro (blah blah) 19:42, 4 June 2010 (UTC)[reply]
Plus, those really aren't the symptoms you'd expect with mercury poisoning. -RunningOnBrains(talk) 16:31, 6 June 2010 (UTC)[reply]
I swallowed a fresh amalgam filling at the dentists while a teenager. I subsequently got a Master's degree so I hope and assume it had no effect on me. I do not recall felling ill or sick. 92.24.185.225 (talk) 13:26, 6 June 2010 (UTC)[reply]

Technological family tree.

Partly a follow on from an earlier disscusion...

Has anyone here come across (in published sources) the idea of a technological family tree?

This being a diagram illustrating which technologies or discoveries are required for a particular technological development to occur.

Technology trees are used in the computer game Civilization and it's sequels and was wondering if these had been used in the academic world as well?

Sfan00 IMG (talk) 13:43, 4 June 2010 (UTC)[reply]

It's not really academic, nor does it involve a formal graphical "tree", but I'm reminded of James Burke's interesting series Connections and its sequels, which traced (some of) the developments that were necessary for particular bits of modern technology to arise. Deor (talk) 14:13, 4 June 2010 (UTC)[reply]
Burke's website here may also be of interest. Ghmyrtle (talk) 15:11, 4 June 2010 (UTC)[reply]
As you can see from his website, though, Burke would be the last person to describe technological advancement in that way as he's spent his professional life illustrating that technology in fact gets developed through all kinds of weird happenstances. He would (and is trying to) make a web, not a tree. Matt Deres (talk) 16:28, 4 June 2010 (UTC)[reply]
Well, the technology trees in Civilization are actually more complicated webs, as I recall. They are greatly simplified from real life, of course. --Tango (talk) 17:03, 4 June 2010 (UTC)[reply]
General histories of technology commonly note breakthrough technology which is the key to some later invention. A powerful compound electromagnet with many layers of insulated wire (Joseph Henry, 1827)) was essential to a practical electric telegraph relay and sounder(one which would function over miles of wire under all conditions), so Morse could not have introduced the practical telegraph before 1827. The phonograph (able to record and play back intelligible speech and music) was invented in 1877, but no breakthrough technology came along just in time as was the case for the Morse telegraph. It could have been built many decades earlier, or as early as good machine tools were available, perhaps in the 1770's, had someone chosen to mechanically impress concentrated sound vibrations on a medium such as wax, tinfoil, lacquer or even soft wood or clay. Edison (talk) 18:30, 4 June 2010 (UTC)[reply]
I think instead of a tree, it would be a directed acyclic graph with all edges pointing in the direction time is flowing. --Rajah (talk) 01:07, 5 June 2010 (UTC)[reply]

UPDATED -- Houseplant identification please... -- UPDATED

Unfortunately, I haven't a picture. In several stores around China I've seen a small houseplant for sale. It has one very large seed - almost as big as a golf ball - that is dark green and split down the middle from which a stem and a few leaves grow. I can't describe it better than that, but if you've ever seen one that should be enough. I've tried all manner of google searches and haven't hit on a picture. Can anyone out there help me? 91.216.105.31 (talk) 13:50, 4 June 2010 (UTC)[reply]

Are you sure it's a seed, and not some form of bulb? Sfan00 IMG (talk) 13:59, 4 June 2010 (UTC)[reply]
If the seed was red, then it could be a lychee which is a fruit, but I doubt if they are grown as house-plants. CS Miller (talk) 14:15, 4 June 2010 (UTC)[reply]
I've grown avocado seeds before, and the description of the seed fits the bill here. --TammyMoet (talk) 14:48, 4 June 2010 (UTC)[reply]
OP here, on a different IP. I'm sure it's not avocado (I've got one of those growing right now) - this seed is a dark forest green color and smaller than that. It has 2 clear hemispheres. I'm also sure it's not a bulb. If all else fails, I'll try and get a picture on Sunday for you all to look at... no chance before then though. 61.189.63.191 (talk) 15:40, 4 June 2010 (UTC)[reply]
Ok, I went and bought one. http://www.drewsjournal.com/JUNK/mystery-plant.jpg Now someone please help me figure out what it is! My original description stands - golfball-sized seed dark green in color, clear hemispheres. The pot pictured fits in the palm of your hand, so maybe 3" across. Too small and wrong color for an avocado, though it certainly share similarities with that plant. Anyone? 91.216.105.21 (talk) 13:33, 5 June 2010 (UTC)[reply]
It looks to me like some kind of nut, see if this page helps any. Ariel. (talk) 03:32, 6 June 2010 (UTC)[reply]

Oil floating under water

News reports say there are "huge plumes" of crude oil from the leak in the Gulf of Mexico which are floating around below the ocean surface. [17] lists densities of various forms of crude ranging from 790 to 973 kg/cubic meter. compared to 1021.98 for "ocean water at 77 F." Tony Hayward of BP claims that"oil has a specific gravity that's about half that of water" per that news story, quite different from the s.g. values cited from the online source. In either case, what force would prevent the less dense crude oil from rising to the surface, or is it uniquely dense, or is the water in the Gulf uniquely less dense? Consider how much force would be exerted by 1 cubic meter of crude at the weakest assumption of 973 density for crude, when it was submerged in seawater at density 1022 kg/cubic meter? Would it be simply the mass difference of one cubic meter (49 kg) times 9.81 m/sec2 or 480.7 Newtons (108 pounds force)? Does the density of the water change more than the density of crude with depth or temperature? Each barrel of oil (.159 cubic meter) should require about 17.1 pounds force to hold it below the surface. If it were in 1 ml droplets or smaller would it somehow not tend to rise? Is it supposed to float under heavier liquid from surface tension/force of habit/? Edison (talk) 16:32, 4 June 2010 (UTC)[reply]

My understanding is that the oil is mixed with gas, which blows it apart into droplets after it escapes. Just as tiny droplets of water may remain suspended in air for a long time, tiny droplets of oil may remain suspended in water for a long time. It's basically what happens when you mix the vinegar and oil to make salad dressing. Looie496 (talk) 16:53, 4 June 2010 (UTC)[reply]
I tried as an experiment releasing 0.1 down to 0.01 ml droplets of motor oil (density about 888 kg/m3 below the surface of a basin of (fresh) water (density about 998 kg/m3. The tiny drops tended to stick to the hypodermic needle, but when I shook it a bit they darted rapidly up to the surface of the water. How small are the "plume" droplets supposed to be, and what principle of physics or chemistry would keep them from rising? So far it sounds like "force of habit." Edison (talk) 16:56, 4 June 2010 (UTC)[reply]
I think it's a result of the dispersant they've been adding to the oil. It causes the oil to form droplets rather than a layer, and those droplets end up suspended in the water. I don't know the details. --Tango (talk) 17:01, 4 June 2010 (UTC)[reply]
If the droplets are down in the "few micron diameter" size then maybe they are in fact rising but some frictional effect of moving the water apart impedes the rise to a slow rate and some surface tension effect impedes their coalescing into larger drops with a greater terminal velocity of rise. Edison (talk) 17:06, 4 June 2010 (UTC)[reply]
Yes - upward force is proportional to cube, 'drag' proportional to square - so smaller drops should accelerate faster slower - though I expect there's a peak velocity in water as there is in air. Also viscosity
Don't think any surface tension effect would prevent coalescing though - unless when as a micelle
Perhaps there are down currents in that area that keep the drops underwater for longer ???87.102.32.39 (talk) 19:18, 4 June 2010 (UTC)[reply]
Keep in mind too that the crude is coming out pretty hot so it may also be partly fractionating, so the light ends will separate and head more quickly to the surface. The s.g. of vacuum tower bottoms is 0.97 to 1.03.[18] Admittedly, that is the very heaviest fraction. Franamax (talk) 17:19, 4 June 2010 (UTC)[reply]
If it so hot that it is "self refining" in a fractionating or cracking process, then why is it simultaneously so cold that it freezes up and clogged the removal dome? And the sludge left in a refinery (if that's what "vacuum tower bottoms " is) is still far lighter than seawater. For the stuff to stay in an emulsified blob below the surface, the blob would have to have a higher density than the water, or the entire column of water would have to be part of the emulsion. Edison (talk) 17:29, 4 June 2010 (UTC)[reply]
First of all, the crude is what's hot, it's the ocean water that is very cold, it is mixing with the gas portion of the outflow to form gas hydrates, that is what is clogging the containment domes. Second, the MSDS I linked for bottoms (it's only sludge when it's really hot, it's asphalt when it cools) says the specific gravity is 0.97-1.03, this [19] shows seawater wirh a s.g. of 1.025, so are you saying 1.025 >> 1.03? Or have you done all the lookups for temperature and pressure and you have a derivation for "far lighter than"? Lastly, all crude oil is "self-refining", that's why it stinks. The lighter fractions are separating preferentially and the benzene is making you dizzy. Crude oil is not one "thing", it is a mixture of many different compounds, each with a different partial pressure. There is certainly no cracking process at work, but the light ends will definitely get to the surface (and evaporate) faster than the heavies. That's why they've been discussing the problem with burning-off once the slick has been floating around more than a few days. Franamax (talk) 03:56, 5 June 2010 (UTC)[reply]
Press reports, such as one from the New York Times say one plume is "3 miles by 10 miles by 300 feet," which would be 7.5 x 10 11 cubic feet, which would be 133 x 109 barrels of oil. The max leak rate estimate is 80,000 barrels a day, or about 3.5 million barrels total so far. Thus there appears to be a gross overstatement of the actual mass of oil involved in the plumes. A scientist states it might have the "consistency of thin salad dressing" in places, fostering the impression that far more mass of oil is in the plumes than seems likely. Edison (talk) 17:44, 4 June 2010 (UTC)[reply]
The media can also be quick to jump on a sensational story and slow to admit error. The laws of physics are hinting to me that this is what is going on. Googlemeister (talk) 18:26, 4 June 2010 (UTC)[reply]
Two things - when the Times gives the size of a plume, they're likely to be describing the volume in which the concentration of oil is appreciably higher than zero — not the volume of pure oil. Compare with coverage of the 2010 eruptions of Eyjafjallajökull, where our article discusses an ash plume more than 8 km high. The ash is there, it's visible, it's definitely occupying that volume — but no one assumes that there's a solid inverted cone of ash more massive than the volcano beneath; there's air mixed in. Indeed, the vast majority of the ash plume is air, just as the vast bulk of these oil plumes is seawater.
A second point that the ash also illustrates nicely is how long it takes for fine particles to settle. Air and volcanic ash particles have a much greater difference in density than oil and water, and air is a far, far less viscous medium than water — but volcanic ash still managed to stay suspended in the atmosphere for days and travel for hundreds of miles. I applaud your experimental spirit in your motor oil and water test, but if the oil is being forced out of the borehole and is undergoing turbulent mixing, then the droplets produced – at least some fraction of them – are apt to be significantly smaller than the 10-100 microliter volumes you tested. Using the creeping flow approximation to calculate terminal velocity, and assuming a density difference of 200 kg/m3 between oil and water one can estimate the rate at which oil droplets will rise to the surface. A sphere 1 mm in diameter (which is quite distinctly visible, though it only has a volume of about 0.5 microliters) would have a terminal velocity of just 3 millimeters per minute; the effect of its bouyancy would in most cases be dwarfed by the effect of rising or falling ocean currents. At that speed, droplets deposited at the borehole depth of 1500 meters would take about a year to reach the surface. TenOfAllTrades(talk) 20:11, 4 June 2010 (UTC)[reply]
One of the biggest sources of error here is accurately estimating the density of the oil that is coming from the reservoir. It is at least plausible that some of the oil in the Gulf of Mexico is on the order of 10° API - in other words, heavy oil - in which case it is not less dense than water. Deep below the sea surface, there is not much turbulence, but as the plume jets its way out of the bore-hole (or some other place where it is leaking), it is probably spraying out in a very turbulent way. This may complicate its path to the surface. If the pressure were low, the plume would rise as a laminar flow to the sea surface (where wave action would turbulently mix it in the top few feet of water). But since the borehole pressure was very high (enough to cause a blowout), it is hard to predict exactly what happens as the oil contacts the open ocean at the sea bottom (or other leak location). As has been pointed out above, the situation is further complicated because the fluid that is leaking from the reservoir is a mixture of crude oil, natural gas, sediments, brine, drilling mud, drilling crud, and other impurities. Nimur (talk) 19:17, 4 June 2010 (UTC)[reply]
Heavy crude oil describes that product as bitumen or oil sand. Is there any evidence that was in the deposit they drilled into, or that it is found in that area of the Gulf? It sounds like heavy crude does not flow well in general. Edison (talk) 20:20, 4 June 2010 (UTC)[reply]
[20] "What we're doing is we're taking a highly pressurized solution, and we're exploding it out of the sea floor under high pressure and high velocity."
expand assuming the oil is tar+oil+gas then at lower pressures the gas will boil off .. pressure increases with depth - it's possible therefor - that as the mixture rises further de-gassing (of methane/ethane etc) happens. It's possible again that this degassing is sufficiently violent (especially if it becomes supersaturate with gas before boiling) that this loss of gas causes the droplets to become yet smaller, again reducing the speed at which they rise. It's also possible that this doesn't happen or the effect is negligable.
I didn't actually see anywhere a claim that the liquid is heavier than water though. Density of 0.8 - 0.9 cc takes quite a long time to rise hundreds of feet - and there may be strong lateral currents.
Did you misread the CEO ? :

Mr. TONY HAYWARD (Chief Executive Officer, BP): There's no evidence of (that) .. (the) oil has a specific gravity that's about half that of water. It's very difficult for oil to stay in the column. It wants to go to the surface because of the difference in specific gravity.[21] [22]

As regards the density vs. depth question compressibility is the thing you want this compares toleune and water (tolune is ~twice as compressibile at 8.94 x 10-4 MPa-1) - so maybe there is something in this idea.. but it's a very small figure (no idea what the pressure is down there) There should be better tables somewhere on the internet.87.102.32.39 (talk) 19:56, 4 June 2010 (UTC)[reply]

87.102.32.39 (talk) 19:27, 4 June 2010 (UTC)[reply]

Hayward seemed to be trying to say that the oil has a low specific gravity and that there was no evidence of plumes (despite the fact that he was speaking in a nonsentence which could be taken to mean the opposite). Edison (talk) 20:22, 4 June 2010 (UTC)[reply]
Uh - yes - I think they mistyped in the link is used (corrected it) - looks like they've joined two sentences together - but it does look like he was saying oil has half the density of water. (The english have acquired the bad habit of lying/stretching the truth in the last decade - I know . live there.)..87.102.32.39 (talk) 20:48, 4 June 2010 (UTC)[reply]
Several news sources ([23] [24] [25]) have used the terms "heavy crude" and "tarballs". It is not clear whether this terminology is scientifically accurate or media hype. Heavy oil is definitely known to exist in the gulf, and indeed is the primary constituent of the geologically similar Orinoco Belt. Biological activity in the deep subsurface and the origin of heavy oil (Nature, 2003), states: "Biodegraded oils also represent a significant fraction of the petroleum in conventional oil reserves and will be common among future oil discoveries likely to be made in deep-water areas of the world (for example, the Atlantic margin basins of Africa, South America, Canada and the Gulf of Mexico)." Reservoir geochemistry of South Pass 61 Field, Gulf of Mexico: compositional heterogeneities reflecting filling history and biodegradation (Organic Geochemistry, 1996) has tables of geochemical analysis. A quick look through some of my Oil & Gas Journal archive has a bunch of market report stories on "heavy" crude in Gulf of Mexico - I'm looking at a February 2010 issue talking about 14 and 17° oil production in Louisiana. Those would still be lighter than water, though. Unfortunately, the exact chemical makeup of the Macondo is pretty intensely "company confidential" and I'm sure only privileged members of BP's production team know what's coming out of the reservoir. Nimur (talk) 20:56, 4 June 2010 (UTC)[reply]
John Ebdon, an Emeritus Professor and chemist at Sheffield University said in a letter in Friday's Guardian (can't find a link): "...True, seawater has a specific gravity of about 1.03g/cubic cm whereas that of crude oil is about 0.8g/cubic cm ... but significant fractions of crude oil consist of asphaltenes and other complex substances with specific gravities around 1.4g/cubic cm ... these heavier fractions will almost certainly separate and so be retained at depth." --Tagishsimon (talk) 04:11, 5 June 2010 (UTC)[reply]
The improvised refinery which fractionates the oil to create the asphalt/bitmen submarine is of dubious plausibility. Asphalt of dubious 1.4 density "retained at depth" would not wash up on Pensacola Beach and would not be much more of an issue than if it had not been brought up by drilling. The media creates spectres of "plumes" of oil many orders of magnitude greater in mass than could possibly exist. Edison (talk) 04:41, 5 June 2010 (UTC)[reply]

Doorbell Continually Hums or Buzzes

the button is stuck in the contact position. how long till the electromagnet will burn out and the bell unit won't work —Preceding unsigned comment added by Alexsmith44 (talkcontribs) 17:30, 4 June 2010 (UTC)[reply]

Why not just disconnect and repair it, rather than go through the discomfort of waiting for it to burn out. [26] which I suspect you've already read as it's linked to from a page almost identical to your question, gives you a description of how to dissemble a doorbell. Regards, --—Cyclonenim | Chat  17:37, 4 June 2010 (UTC)[reply]


can you just tell me how long till it burns out? i dont have the tools right now. —Preceding unsigned comment added by Alexsmith44 (talkcontribs) 17:50, 4 June 2010 (UTC)[reply]

It depends, but "long" is a good estimate. Think weeks or months, or even years. There is no particular reason why anything should "burn out". --Stephan Schulz (talk) 17:57, 4 June 2010 (UTC)[reply]
A defective doorbell circuit is likely to cause radio, television and celphone interference, which is one good reason to get it fixed. Some doorbell transformers have a thermal protection element which heats up and opens the current , then resloses, providen a spark each time which interferes with electronic communication. The thermal protector often burns out and just arcs continuously, provide ongoing interference. In the US, it is a violation of the FCC rules to create "harmful interference". A shorted transformer could also start a fire, [27]. (Paradoxically, in a great many instances over the years, doorbell wires shorted by a house fire have alerted the family and saved their lives).How long would it take to burn out? I once read an account of a person growing up in an apartment with a defective doorbell circuit. The transformer or chime hummed all the time, 24/7, except when someone pressed the doorbell, at which time it quit humming momentarily, and they knew someone was at the door. Edison (talk) 18:00, 4 June 2010 (UTC)[reply]
Doorbells are often very high-current devices - you may be burning two or five amps if the electromagnet is constantly activated. (Some are lower-power - here's a 30 watt bell). Most bells are not intended to be "energy-efficient", because you want the bell to be loud and momentary - so effectively, they are designed to consume on the order of 50 or 100 watts, (but quickly shut off). If the switch is stuck in contact-position, and you are burning a steady 50 or 100 watts, over a month that will add up) and will cost you in the neighborhood of $5, depending on your electricity price. Nimur (talk) 19:08, 4 June 2010 (UTC)[reply]
That makes me wonder, is there a danger that it was installed with wiring that can't handle constant use? APL (talk) 20:45, 4 June 2010 (UTC)[reply]
Actually, since you mention it - that's certainly possible in older houses. In the UK, there is stuff called "bell wire" that used to be used specifially for wiring up doorbells and those bell systems in big houses that let the lady or gentleman of the house summon servants from the kitchen from any room in the house. That wire was very thin indeed, single-core with enamel paint insulation. Bell wire was the home-experimenters wire-of-choice back when I was a kid because it was cheap and you could get it in any electrical repair store. So it wouldn't surprise me at all if it overheated and burned out in an old property...and that's a fire risk. But I'd expect chunkier wiring in modern doorbells - so it's probably not a problem for a house that's less than maybe 50 or 60 years old. SteveBaker (talk) 04:25, 5 June 2010 (UTC)[reply]

can any of these radio frequencies harm me?

No. APL (talk) 20:45, 4 June 2010 (UTC)[reply]
Really - you can trivially repair this with no more tools than a screwdriver - or (if desperate) a kitchen knife - or maybe even a thin coin! Waiting - on the offchance that it'll just blow up and "fix itself" - is pretty pathetic. Turn off the power, unscrew the bell button or pop open the cover, if the contacts inside look corroded then just scrape off the surface corrosion with your knife/screwdriver - then bend them so they don't quite touch when the button isn't pressed, and do touch when it is pressed. Reassemble the button and turn the power back on - and you're done. It couldn't possibly take 10 minutes to fix. SteveBaker (talk) 23:12, 4 June 2010 (UTC)[reply]
Curiously enough, there was "bell wire" before electricity was in use. Great houses in the 18th century, including the White House in John Adams' Presidency had "bell pulls" to signal servants that someone in a certain room wanted a servant to come. The "bell wire" was uninsulated copper, brass, or iron running to the kitchen or "Servant Central" where a pull on it rang a bell. Edison (talk) 04:31, 5 June 2010 (UTC)[reply]
Yes - I noticed that when I searched Wikipedia for bell wire and found it mentioned in Bell pull. It did surprise me to see the term used that way. I wonder whether these old houses went from physically pulling the wires to passing electicity down them when they converted to new-fangled electrical systems (maybe using an earth as the return path). That would save ripping out the walls to put in the new wires. But the stuff I remember having as a kid in the late 1950's was single-core copper, very brittle and coated with what looks like a brown enamel paint or maybe a dark varnish...that was called "bell wire" too. The electrical projects in "Boy's Own" style books always specified either "Bell wire" for battery-powered things or "Lamp wire" where higher current was required. This definition on Answers.com confirms my memory. SteveBaker (talk) 04:56, 5 June 2010 (UTC)[reply]

The difference between fire and plasma

Take this image for example, it's a projectile fired from a railgun. But of course, there are no explosives involved so that trail behind the projectile is plasma and not fire correct? So is it fair to say that plasma and fire can look the same sometimes, but are two different things? ScienceApe (talk) 18:06, 4 June 2010 (UTC)[reply]

You don't need explosives for flame; just heat, oxygen, and something to burn. --Sean 18:53, 4 June 2010 (UTC)[reply]
(ec) Most of the time, ("day-to-day" interpretation), when you think of fire, you are thinking about two things: (1) an oxidation reaction between oxygen in the air and carbon (or something else) in the fuel; and (2) a gaseous semi-ionized plume of hot air that is incandescent - glowing reddish yellow because it is so hot. The chemical reaction (1) provides the energy and heat, and contributes some chemical constituents to the flame plume, but mostly the glowy part is just the ambient air getting heated up (and indeed, somewhat ionized). It would be fair to call that flame plume a "plasma" - but more specifically, it would be a dense weakly-ionized plasma. In other words, it is more "gas-like" than "plasma-like", in that thermal interactions dominate the behavior, rather than electromagnetic interactions. Typically, in plasma physics, we like to think about "sparse" (i.e., nearly a vacuum) plasmas where the collisions between molecules and ions are very rare. But plasmas can be created in many ways - anything which provides enough energy to ionize a gas sufficiently that its electromagnetic effects are non-negligible. That can be a chemical flame, or any other energy source.
All this being said, let's analyze the railgun photo you linked. Unfortunately, the caption/description is not really very explanatory - it says the gun is "electromagnetic," but that doesn't necessarily mean that there was no explosive (a lot of railguns use a primer charge to create a super-compressed gas in preparation for additional energy). It's also possible that at 2000 m/s, the hypersonic projectile is tearing up its surrounding air so much that the thermal energy imparted to the shock cone is enough to incandesce or even ionize. To what extent is the shock cone ionizing? To what extent does the electromagnetic propulsion system ionize the surrounding air? To what extent does the primer-charge interact with the muzzle blast? How do these parameters affect the muzzle ballistics, kinetics, trajectory, and the energy balance for the projectile? All of these things would be interesting research questions, and I would bet that the NSWC guys are instrumenting the heck out of this apparatus to answer exactly those kinds of questions. But we only have the image caption - without further information, it's not easy to answer definitively. But keep in mind - a plasma is just a gas that has been ionized so much that the electromagnetic effects are dominant, whether it's ionized by energy from a chemical fire, thermal collisions, electromagnetic radiation, nuclear energy, hypersonic shock, or other energy source. Nimur (talk) 18:57, 4 June 2010 (UTC)[reply]
chemisty I've always understood most flame/fire to be primarily a radical reaction - also high energy but not a "full" plasma since many of the entities are neutral. If the flame is hot enough though gas will be ionised.. answer? it's a borderline case depending on T and gases burnt as well.87.102.32.39 (talk) 19:38, 4 June 2010 (UTC)[reply]
(EC)Plasma and a fire can look alike. Both can involve very hot gases. See Plasma (physics), which says that a plasma is electrically conductive because of ionization. But flame and smoke can also be conductive, depending on what the chemical composition is. Edison (talk) 19:01, 4 June 2010 (UTC)[reply]
If you look at the video that this shot was taken from, I think that it appears to be flame, not plasma. J.delanoygabsadds 19:05, 4 June 2010 (UTC)[reply]
I can't tell - but yellow air plasma isn't usual. A plasma should decay without any soot etc. Maybe there was some coating on the 'bullet' that caused the plasma to go yellow?87.102.32.39 (talk) 19:45, 4 June 2010 (UTC)[reply]
You're right, for it to glow that brightly there must be something more than just incandescent air. With a flame, there are tiny particles of soot glowing (if you have complete combustion with no soot then you don't get a yellow flame, you get a dim blue flame that isn't caused by incandescence). There must be something similar in that plume. --Tango (talk) 19:54, 4 June 2010 (UTC)[reply]
This sounds 'clu-ey' [28] "Particle debris ignites as a test slug exits the Office of Naval Research (ONR) 32 MJ (megajoules) Electromagnetic Railgun (EMRG) laboratory launcher located at the Naval Surface Warfare Center Dahlgren Division (NSWCDD)." 87.102.32.39 (talk) 20:04, 4 June 2010 (UTC)[reply]
Also, we have no way of knowing whether the photo is a true-color or false color image. I'm leaning towards a Schlieren photography, Rapatronic camera, polarizing filters, or some other "weird" photographic trickery (not even counting post-production digital color rescaling!) Note that the timestamp has microsecond precision. Such high-speed cameras often are monochromatic. Note how you can see the shock cone - that's a dead giveaway for a shadowgraph or Schlieren system. So, we can't say for sure whether the yellow and green in this photograph are "meaningful" for analysis. Nimur (talk) 21:11, 4 June 2010 (UTC)[reply]
There's an even more clear image of the shock cone at this link (scroll down) [29] 87.102.32.39 (talk) 21:30, 4 June 2010 (UTC)[reply]
Horse's mouth [30] "The flames are from pieces of the projectile disintegrating; the 7-pound slug is jammed so firmly between the rails that when it’s fired, pieces shear off and ignite in the air."
one commenter says "...but the video is slightly out of order... the first shot is of the impact. If you pause the video around 6-7 seconds you can see the projectile only has a plasma trail until it strikes the target." (same link) this isn't quite right the image above is earlier in time than that in which the projectile flys between gun and target.
The linked video has time count in the lower right . so if doubtful you can check for yourself. 87.102.32.39 (talk) 21:49, 4 June 2010 (UTC)[reply]
This image is confusing File:EMRG 070128-N-0000X-001.jpg - apparently showing a very similar moment in time - but a different colour - since the slug is aluminium and the fire is supposed to be from particles from the slug I would assume that the photo is a better colour representation. So why is the video footage (and the frame above) yellow?87.102.32.39 (talk) 22:12, 4 June 2010 (UTC)[reply]

Archimedes principle???

if a body is immersed in a fluid,is the volume of the displaced fluid equal to the volume of the body???......... —Preceding unsigned comment added by Researcher02 (talkcontribs) 19:38, 4 June 2010 (UTC)[reply]

Only if it sinks - this can be demonstrated with a full bathtub, large amounts of polystyrene foam, and some lead shot..
The density of the body must be more than the density of the fluid.87.102.32.39 (talk) 19:42, 4 June 2010 (UTC)[reply]
(e/c) under conventional assumptions (non-viscous fluid, unconstrained and not otherwise subject to compression, non-bouyant, non-dissolvable object, etc), yes. The fluid will conform to the shape of the object (so displacement will not be greater than the volume of the object). note that for hollow objects, displacement will differ according to whether the interior spaces are open or sealed - open spaces will fill, making the displacement equal to the volume of the solid material; sealed spaces will displace both the volume of the solid mass and the volume of the sealed space. --Ludwigs2 19:48, 4 June 2010 (UTC)[reply]
(ec) The volume of the displaced fluid is equal to the volume of that part of the body that is in the fluid. If the object sinks, that will be the whole body. If the object is floating then it will only be part of the body (a part with volume equal to the volume of the fluid that has the same mass as the object). --Tango (talk) 19:49, 4 June 2010 (UTC)[reply]
That assumes that the fluid displaces all the air in the body - if air is trapped (closed-top, open-bottom area or by hairs, etc), then the trapped air has to be counted as part of the body (after compression). CS Miller (talk) 13:33, 5 June 2010 (UTC)[reply]
The article section about this is Buoyancy#Archimedes' principle. Comet Tuttle (talk) 21:09, 4 June 2010 (UTC)[reply]
The OP's question does not concern Archimedes' principle. The OP did not ask about a floating body. Cuddlyable3 (talk) 22:41, 4 June 2010 (UTC)[reply]

Why did my bonfire burn 'blue'?

Last summer, I used a bonfire to dispose of a large quantity of waste wood (mainly laurel branches) in my garden..

I noted that, when started the fire was rather yellow and produced a lot of smoke, but as it got going it smoked a lot less and changed from a yellow flame to a more blueish and less visible one...

What might be the cause of this?

In addition, a previous bonfire in a previous year apparently managed to make a small quantity of charcoal, although I'm not entirely sure how... The same bonfire for whatever reason was also able to be 'relit' merely by placing suitably dry fuel on the ashes the following morning, which came as a quite a suprise...

Anyone care to suggest a possible explanation for the charcoal production? or for the 're-light' capability? 62.56.112.251 (talk) 20:19, 4 June 2010 (UTC)[reply]

It's very common for a wood fire to create coals that stay burning hot overnight, in some cases they can continue to burn for days. That article describes them better, but briefly: after the outside of the wood burns, and the more volatile componants of the wood have escaped and burned, the heavier carbon based solids in the innermost parts of the wood stick around because they haven't been properly exposed to the air, but at the same time they're not volatile to evaporate through the hot wood. Usually these componants burn very slowly making hot coals. (These are good for cooking over, because of their even heat.) If they go out before they're entirely spent I suppose you'd have charcoal. (Which does not come from true coal. A lot of people are confused on this point.)
Hot coals can stay hot for a suprisingly long time because they're covered in a layer of light, fluffy ash that both insulates them and prevents very much oxygen from reaching them. That way they don't cool off, but they also don't burn up very quickly. APL (talk) 20:39, 4 June 2010 (UTC)[reply]
Indeed - a fairly large bonfire will usually have hot embers left for several days. --Tango (talk) 21:12, 4 June 2010 (UTC)[reply]
Follow on question, Would this 'coals' method be exploitable for low-level metalworking? Sfan00 IMG (talk) 21:55, 4 June 2010 (UTC)[reply]
Did you mean could the charcoal you made be used for metalworking? I see no reason why not - it was used in the past,as the article charcoal says. 87.102.32.39 (talk) 03:21, 5 June 2010 (UTC)[reply]

Hmm. Have you seen She? --Trovatore (talk) 20:42, 4 June 2010 (UTC)[reply]

I would think that at first the oxygen supply was limited. After a while it started burning more completely, producing the hotter blue flame and less smoke. --Chemicalinterest (talk) 20:54, 4 June 2010 (UTC)[reply]
The yellow flames are incomplete combustion. The blue flames are complete combustion. If everything burns cleanly then there are no soot particles to glow red-hot, which is what gives the flame its colour. --Tango (talk) 21:12, 4 June 2010 (UTC)[reply]

June 5

What is the reduction potential for this reaction: 2 NO3- + 12 H+ + 10 e- ←→ N2 + 6 H2O Thank you. --Chemicalinterest (talk) 00:28, 5 June 2010 (UTC)[reply]

from [31]
NO3- > NO 0.957
NO > N2O 1.591
N2O > N2 1.766
I think you can just add those ~2.2V ? Maybe not - different number of N atoms... Different number of electrons transfered in each reaction.
It's {(0.957x3)+(1.591)+(1.766)}/5 (reason for that formula at Standard_electrode_potential 87.102.32.39 (talk) 01:20, 5 June 2010 (UTC)[reply]
About 1.25V also see here [32] 87.102.32.39 (talk) 01:39, 5 June 2010 (UTC)[reply]
Thanks for your help. (BTW, when you type chemical formulas, it is recommended to put subscripts and superscripts around the appropriate figures, such as NO<sub>3</sub></sup>-</sup>, making NO3-) I don't think it is 5V. Even F2 gas is +2.87 or +3.01. Many times going from step to step is harder than "taking all the steps". Hypochlorite, chlorite, chlorate, perchlorate is an example. Each one has an oxidation potential of about +1.5V to the next one, yet perchlorates are not supremely reactive because the steps are not added up (they are actually the least reactive). +1.25V seems right, a little higher than the reduction potential (+1.23) for oxygen in an acidic environment. --Chemicalinterest (talk) 11:07, 5 June 2010 (UTC)[reply]
Yes the EMF is an average - that comes from the energies of reaction being additive, but the actual EMF's are measured 'per electron transfered' - thus energy change = electrons x voltage
eg for NO3- >> NO (change from V state to II state)
eg NO3- + 4H+ + 3e >> NO + 2H2O + NO (3 electrons)
..the energy change is three times that what it would be if only one electron had been transfered
this equation in fact :ΔGcell = −nFEcell where F is Faraday constant
It's actually similar at a simple level to the formala for potential energy of a charged particle in an electrostatic potential E=QV (or as written in Electrostatic_potential#In_electrostatics UE = qV ) 87.102.43.94 (talk) 11:41, 5 June 2010 (UTC)[reply]

Who first hypothesized/discovered/elucidated gluconeogenesis?

Who first hypothesized or discovered or elucidated gluconeogenesis? Thank you. --Rajah (talk) 00:54, 5 June 2010 (UTC)[reply]

this link mentions some key people (last 2 paragraphs) 87.102.32.39 (talk) 03:17, 5 June 2010 (UTC)[reply]
Thanks, but that's about glyceroneogenesis, which is a subset of gluconeogenesis. The conversion of pyruvate to glucose would be the major component of gluconeogenesis. --Rajah (talk) 06:52, 5 June 2010 (UTC)[reply]
Sorry, I was working quite late - didn't even notice at the time. Please accept my apologies.87.102.32.39 (talk) 10:51, 5 June 2010 (UTC)[reply]
This is not a topic I know anything about, but just for the hell of it I trotted out my Googling skills, and found that the fundamental work on the chemistry was done by A. I. Ringer and colleagues in a series of papers in the Journal of Biological Chemistry in 1913 and the following few years. The physiological background was worked out over the previous few decades, largely in German publications. The state of affairs as of 1913 is reviewed in this paper. Looie496 (talk) 22:33, 5 June 2010 (UTC)[reply]

Brass

Alloys of Copper and Zinc create a how to hop metal brass.lam not work on this solution and HCl solution H2SO4 ? —Preceding unsigned comment added by I love chemistry (talkcontribs) 04:02, 5 June 2010 (UTC)[reply]

Could you please rephrase your question, I don't really get what you're trying to do. 67.170.215.166 (talk) 04:30, 5 June 2010 (UTC)[reply]
Sounds like "I love chemistry" is asking how to separate Cu and Zn in brass using HCl and H2SO4, which sounds a little like a homework problem, though I won't assume, but I recommend checking reduction potentials and noting if any salts will precipitate. SamuelRiv (talk) 05:41, 5 June 2010 (UTC)[reply]
This is original research. I tried dissolving brass in hydrochloric acid but it is very difficult too, it is a rather noble alloy. Do not expect the zinc to come fizzling out, leaving a framework of copper behind. --Chemicalinterest (talk) 11:08, 5 June 2010 (UTC)[reply]
See galvanic series; you can see that it is rather noble (closer to the top than say, aluminum, iron)--Chemicalinterest (talk) 11:15, 5 June 2010 (UTC)[reply]
More so than either copper or zinc - I don't really understand why.87.102.43.94 (talk) 11:27, 5 June 2010 (UTC)[reply]
If the aim is an analysis of Cu and Zn in brass then this link gives the standard proceedure [33] or this 87.102.43.94 (talk) 11:25, 5 June 2010 (UTC)[reply]
If you want to know how to make brass there is an interesting but old manual here [34] eg "Put 4 1/2 lbs. of copper into a crucible, expose it to heat in a furnace, and when perfectly fused add 1 1/2 lbs. of zinc. The metals will combine, forming that generally used alloy called brass." there are recipes for other brasses too. And a more up to date list at Brass#Brass_types ? 87.102.43.94 (talk) 12:09, 5 June 2010 (UTC)[reply]

Asa Gray and John Edward Gray

I'm planning on editing a few articles on nineteenth century biology and I've come across a question I can't answer. Is there a relationship between Asa Gray and John Edward Gray? I know one is British and the other American, but they both seem to be biologists during the same time period. I'm most interested in if they are related biologically and if they there are any primary sources mentioning the relatedness of both of them. Thanks a bunches :) Peter Napkin Dance Party (talk) 07:27, 5 June 2010 (UTC)[reply]

The WP:RD/H may be a better place. --Chemicalinterest (talk) 11:09, 5 June 2010 (UTC)[reply]
Here'll do. It's not looking very likely, or at least not in any meaningful span of generations. AG is described as being "of Scotch-Irish descent" [35] whilst JEG's grandfather was a London seedsman who died without inheritance [36]. I'd question how much migration of "Scotch-Irish" into the London area was going on in the mid or early C18 ... not very much. All in all, has the feel of a coincidence of surnames and nothing more. --Tagishsimon (talk) 13:02, 5 June 2010 (UTC)[reply]

Brass (30% Zinc and 70% Copper)

How to Brass (alloy of 30% Zinc and Copper 70%) not destroyed in the dense environment axid Chlohydric, axid in solution and concentrated sulfuric (H2SO4)and water Amonia (NH3). No surface coating of the alloy, only add other elements to increase resistance axid, but no change yellow color of the alloy elements nay.Neu you know is right, then please just help with!--I love chemistry (talk) 13:40, 5 June 2010 (UTC)[reply]

Your question does not seem very clear. Do you want to know why acids don't attack brass? --Chemicalinterest (talk) 13:41, 5 June 2010 (UTC)[reply]
If that is your question, then my explanation is that: On the surface, there is a mixture of copper and zinc. The small amount of zinc corrodes, leaving only copper on the surface (explains why few bubbles appear on surface when immersed in acid). The copper is resistant to acid attack (see standard reduction potential for explanation of oxidation and reduction, also see redox), which prevents further corrosion. Brass is also resistant to corrosion by the oxidizing properties of copper salts. Again, the small amount of zinc on the surface dissolves, plating the brass with a durable coating of copper metal.
In short, when the zinc dissolves, the brass is essentially a copper-coated strip of brass. --Chemicalinterest (talk) 13:48, 5 June 2010 (UTC)[reply]
Actually in ammonia the copper actually dissolves. In air, copper forms a protective coating of copper(II) oxide. That dissolves in ammonia to form a complex similar to tetramminecopper(II) sulfate. A new coating forms and dissolves, eating away the copper. This happens very slowly though. It also happens in acid, albeit very very slowly. --Chemicalinterest (talk) 14:10, 5 June 2010 (UTC)[reply]
If you wanna dissolve brass, you could stick it in nitric acid, and that will dissolve it easily. (Getting the nitric acid might be hard, though -- it's a restricted chemical because it can be used to make bombs.) FWiW 67.170.215.166 (talk) 01:42, 6 June 2010 (UTC)[reply]
Simpler way; mix 3% (household) hydrogen peroxide and household muriatic acid in about a 1:1 ratio. That should dissolve brass, the H2O2 functioning as an oxidizer and the HCl dissolving the oxide produced so the peroxide can continue oxidizing the metal. --Chemicalinterest (talk) 12:05, 6 June 2010 (UTC)[reply]
This might also be a good way to make a binary chemical weapon: H2O2 + 2HCl → 2H2O + Cl2. Don't try this at home (at any rate, not unless you do this in a fume hood and while wearing a gas mask). And if you do get the results that I've predicted, Mr./Ms. I-Love-Chemistry, don't blame me. 67.170.215.166 (talk) 00:51, 7 June 2010 (UTC)[reply]
Chemicalinterest, didn't your chemistry teacher tell you not to mix halides with strong oxidizers? 67.170.215.166 (talk) 01:09, 7 June 2010 (UTC)[reply]

Information on Planting lilies in green house

Dear sir I am asking for information about planting lilies in green house

  1. first steps in soil ( compost & per light )
  2. soil ( EC & PH )
  3. fertilizer program ( 2 weeks 2 – 1 – 1 & 6 weeks 1 – 1 – 1 )

Spatially fertilizer in ( first – medal – end ) life
I am from medal est. ( UAE ) & I will planting on 1st October <E-mail address removed> thank you
hamid —Preceding unsigned comment added by 94.59.56.184 (talk) 13:52, 5 June 2010 (UTC)[reply]

Hi Hamid, we don't respond to queries by email so I've removed your address. Brammers (talk/c) 18:39, 5 June 2010 (UTC)[reply]

You question does not say which type of lily you are going to grow. You can see from the article that there are many species and some require special conditions to succeed. Some lilies require deeper planting because they develop roots from the stems, others need a deep soil with a 'woodland' shade. Of course many will grow in a nice soil with compost, some slow-release fertilizer and regular watering. My experience with lilies is that you don't need a complicated watering/fertilizing programme. If you are going to grow them commercially then you will need specialised information. Richard Avery (talk) 07:32, 6 June 2010 (UTC)[reply]

Deepwater Horizon valves

Hi. Could a piston valve assist in one of the procedues used to cap the flow of oil or to redirect it into a ship? Thanks. ~AH1(TCU) 14:38, 5 June 2010 (UTC)[reply]

A Piston valve could indeed help - in fact the automatic shutoff valves that failed (the Blowout preventers) causing the problem are (or can be) a type (or variation of) of piston valve (in that the closing motion is linear).87.102.43.94 (talk) 15:05, 5 June 2010 (UTC)[reply]
A Gate valve has some similarities in design and would be another good choice for closing of a wide pipe.87.102.43.94 (talk) 15:13, 5 June 2010 (UTC)[reply]
Has anybody suggested this idea to BP? ~AH1(TCU) 15:22, 5 June 2010 (UTC)[reply]
They've been cutting through the pipe to get a neat end on it - I would expect that their next step is to fit such a valve. 87.102.43.94 (talk) 15:27, 5 June 2010 (UTC)[reply]
Update: our folks have fitted a new BOP just the other day, but we're keeping it partly open for now to keep it from getting jammed with clathrates. Right now only about 1/3 of the oil is being captured -- we'll be screwing it down to about 90% capture over the next few days, or to as low a flow rate as clathrate formation permits -- but we'll have to keep it open a crack until the relief well is completed. 67.170.215.166 (talk) 01:48, 6 June 2010 (UTC)[reply]
ok I'll bite.. if it gets jammed with clathrates does that not stop the oil flow - is that not a good thing?77.86.124.76 (talk) 14:59, 6 June 2010 (UTC)[reply]
You're wrong, 77 IP -- right now, the valve package is set up to divert part of the oil to a recovery ship, thus keeping the pressure down; if the thing gets clogged up with clathrates, it'll block that diversion and cause a pressure rise that could make the whole setup blow out again. 67.170.215.166 (talk) 00:58, 7 June 2010 (UTC)[reply]
Clathrates, as in clathrate hydrate and methane clathrate? Could that release greenhouse gases into the atmosphere? ~AH1(TCU) 01:36, 7 June 2010 (UTC)[reply]

Nature/environment/biotope

Hi there! What's the difference between nature, natural environment and biotope? These three (especially the first two) concepts seem extremely similar but since there are three articles I suppose that there is some kind of difference. Could anyone help me to clarify this? /Natox (talk) 14:51, 5 June 2010 (UTC)[reply]

Nature and natural environment are almost synonyms. Nature is an older, looser term that refers to anything not man-made; natural environment is a bit more technical (though less technical than ecosystem or biosphere). Biotope is a particular type of natural environment. --Ludwigs2 15:17, 5 June 2010 (UTC)[reply]
(edit conflict - very similar answer)
Nature has multiple meanings (dictionary) one of which is synonymous (has the same meaning) as "natural enviroment".
As for "biotope" it usually refers to a specific type or area of natural enviroment - eg a "tropical biotope" or a "industrial wasteland biotope", it can even refer to small areas such as the types of biotope a bacteria might live in eg "gut biotope" , or even "fridge biotope" - the meaning and usage is quite similar in usage to ecosystem, but they are different and do have technically different meanings. ie an ecosystem will have many different biotopes within it - eg tropical rainforest ecosystem contains river biotope, stream biotope, stream bank biotope, leaf litter biotope, tree canopy biotope, mud biotope and many other biotopes.87.102.43.94 (talk) 15:20, 5 June 2010 (UTC)[reply]
Thank you both for good answers. This raises another question: How come there are two articles, natural environment and nature, if there is hardly any difference? They seem to be on the exact same subject. /Natox (talk) 15:39, 5 June 2010 (UTC)[reply]
Yes - I think that's a mistake - it looks like they should be (WP:MERGE) merged - at least I can't see any reason for two articles.87.102.43.94 (talk) 15:48, 5 June 2010 (UTC)[reply]
I've suggested that they be considered for merging.87.102.43.94 (talk) 15:54, 5 June 2010 (UTC)[reply]

Energy efficient of distillation

Assuming that any enthalpy change in the process is negligible, can the energy efficiency of distillation be arbitrarily close to 100%? Is there a reason why a certain minimum amount of energy will always be lost (i.e. become unavailable) in the process? --173.49.77.55 (talk) 16:14, 5 June 2010 (UTC)[reply]

(There needs to be an real valued enthalpy of vaporisation for distillation to be possible)
When you say efficient I assume you mean that 100% of the energy supplied to the system is converted into liquid boiling? If so yes it can approach 100% with effective insulation - there will always be a latent heat of the apparatus (ie start up) - ignoring this ie using continuous distillation the figure can approach 100% but not reach it because perfect insulation doesn't exist.
However if you say that the liquid for distillation is supplied 'cold' and do not include energy required to raise the T of the liquid (ie specific heat) then this will cause a less than 100% figure in the efficiency figure.87.102.43.94 (talk) 18:50, 5 June 2010 (UTC)[reply]
I was thinking about water purification by distillation when I asked the question. By negligible enthalpy change, I mean that you can ignore the energy that is inherently required to separate the impurities from the water, such as enthalpy of solvation. To simplify the problem, just consider the case when you're distilling pure water and condensing it back. Some of the energy used to boil the water can be recouped using a heat exchanger, such as a countercurrent heat exchanger. If the process is 100% energy efficient, the effluent water is at exactly the same temperature as the incoming water, and no incremental energy is needed to process an additional unit volume of water. That seems not possible in practice. Hence the question. --173.49.77.55 (talk) 23:46, 5 June 2010 (UTC)[reply]
I think the issue here is of getting energy to flow one way (from condenser to boiler). I think what you are describing is (the same as) either using the boiling water as condenser fluid or connecting the boiling water and condensor fluid so that they are at the same temperature. Additionally for the 100% efficiency the whole apparatus would have to be insulated -this means that the receiver would be at 100C as well - this would make the whole apparatus in thermal equilibrium - so there would be no net liquid flow.
To get net flow the condensor would need to be at a lower temperature than the boiling liquid - for this a heat pump would be needed - I'm not sure whether this heat pump can be 100% efficient.87.102.43.94 (talk) 12:58, 6 June 2010 (UTC)[reply]
I think theorectically you are right, eg if the whole apparatus is at 100C and a heat pump (see heat pump and related topics) is used to move energy from the reciever to boiler then then theoretical efficiency is infinite.. But it still requires a finite energy input for anything to actually happen. Also what might need to be considered is the need for energy input to prevent a 50:50 distribution of liquid in receiver/boiler coming about be the two being in effective equilibrium.87.102.43.94 (talk) 13:06, 6 June 2010 (UTC)[reply]
start again simple explanation - The idea of the distillation is to transfer a liquid from A to B , both at the same temperature. The energy required to do this is 0. But to actually get anything to happen there needs to be a non-zero energy difference between the two states (start state and finish state) - a non zero amount of energy will need to be supplied constantly to maintaine an energy differential (A>B) while the process is proceeding.87.102.43.94 (talk) 13:58, 6 June 2010 (UTC)[reply]
The energy required can approach zero, but when it is zero nothing happens.87.102.43.94 (talk) 14:00, 6 June 2010 (UTC)[reply]
If the purpose of the destillation is to desalinate water, you must take into account that the vapor pressure of salt water is lower than the vapor pressure of sweet water, so without doing work on the system the process will go in the wrong direction. The salt water must be warmer in order to increase to vapor pressure above that of sweet water. Bo Jacoby (talk) 21:29, 6 June 2010 (UTC).[reply]

Beer question

Why does beer taste better out of a glass bottle than a can? I'm mainly talking about bitters and ales here, I don't tend to drink many macrobrew lagers. 188.221.55.165 (talk) 17:48, 5 June 2010 (UTC)[reply]

Glass doesn't taint the liquid inside it. I'm sure someone will contradict me, but even if cans are lined with plastic, you can taste the tin, which must mean it has tainted the contents. Also you can't put real ale into a can because it will have to be pasteurised during the canning process, which means it loses its real ale status. Filtering and pasteurising beer will also alter the taste. You can get real ale in a bottle, and you can have filtered and pasteurised ale in a bottle. Personally (and I will be drummed out of CAMRA for saying this) I will gladly drink a bottle of beer even if it's not real ale, because the quality can be almost guaranteed. --TammyMoet (talk) 19:17, 5 June 2010 (UTC)[reply]
For one thing, it's often cheap beers that are distributed in cans. To make a fair comparison, you'd need to look at the same brand of beer in both cans and bottles. From a manufacturing standpoint, canned beers are clearly superior: it's easy to seal a can, and no light can get in. I would suspect that both the glass and can are relatively neutral in regards to taste, and that it's much more likely to be the feel of glass vs. metal that makes a difference in terms of enjoyment (and note that beer cans are aluminum, not tin, as TammyMoet incorrectly asserts). Many of the people online suggest that beer should always be drunk from a (glass) cup, whether it comes in a can or a bottle. See also [37] for one among many opinions on the matter. Googling "beer can vs bottle" will bring up tons of other opinions on the matter. Buddy431 (talk) 22:15, 5 June 2010 (UTC)[reply]
'Tin' is used as a synonym for 'can' in the UK. I doubt Tammy is asserting anything about the composition of the can/tin, only that it can be tasted. And indeed it can, although I suspect that is mostly the smell from the metal on top, particularly the exposed surface you've just created in opening it. I wonder if the same effect is evident when the beer is decanted before drinking. 86.164.69.239 (talk) 22:48, 5 June 2010 (UTC)[reply]
Agreed to the small comment above; beer in a can is different to beer in a bottle. Try Budweiser in a beer can and bottle, you'll notice a slight difference. The tin taints the beer slightly. Regards, --—Cyclonenim | Chat  23:29, 5 June 2010 (UTC)[reply]
BTW, there is no tin in tin cans. They are made of steel or aluminum. And for a proper test pour the beer into a glass before drinking it. Ariel. (talk) 03:37, 6 June 2010 (UTC)[reply]
To clarify, I wasn't asserting cans were made out of tin, but in the UK and associated countries, cans are known as "tins". When an Australian refers to a "tinnie", she is referring to a can of beer. --TammyMoet (talk) 08:15, 6 June 2010 (UTC)[reply]
The UK/Aussie usage is based on the fact that cans were originally lined with tin, and were called tin cans. Often, the "can" was just dropped, and hence the popular usage of just calling any metal container a "tin". --Phil Holmes (talk) 11:54, 6 June 2010 (UTC)[reply]
The OP needs to first test the assumption by doing a blind taste test among several friends. Its an easy thing to do: Find friends that claim they can tell the difference between bottled and canned beer, obtain cans and bottles of the same brand, then pour each into identical glasses, and see who can tell the difference. --Jayron32 03:14, 7 June 2010 (UTC)[reply]

Vegetable cooking oils - two groups of questions

I never fry food but I do drizzel pasta with a small amount of cold oil. Questions 1) Trans-fat in oil. This http://www.nal.usda.gov/fnic/foodcomp/search/index.html says that canola/rapeseed oil is 0.395% trans-fat, and sunflower is 0.219% trans-fat. It does not give any figures for the other oils I have looked up. How or why is the trans-fat in there? Where can I find reliable figures for the amount of trans-fat in other vegetable oils?

Question 2). I have just bought a litre of "vegetable oil" from a supermarket in the UK. What is it likely to be, more specifically? Thanks 92.24.182.48 (talk) 18:41, 5 June 2010 (UTC)[reply]

(2) It's likely to be rape oil. 213.122.16.179 (talk) 19:28, 5 June 2010 (UTC)[reply]
agree - there's a list at Cooking_oil#Types_of_oils_and_their_characteristics some of which are not vegetable, some of which are too flavoured to be used, but quite a few can and are used in a vegatable oil.
There's a more specific list [[38]] it's quite likely to be a mixture as well.87.102.43.94 (talk) 20:26, 5 June 2010 (UTC)[reply]
Note that Canola oil is merely a type of rape oil. I think that "vegetable oil" is likely to be a mixture consisting of whatever's cheapest/easiest to obtain at the time of manufacture. Buddy431 (talk) 22:08, 5 June 2010 (UTC)[reply]
Well, it's not "merely a type" of rapeseed oil — it's the only type, as far as I know, that's considered fit for human consumption. Rapeseed oil other than Canola has too much erucic acid and is considered toxic (maybe not very toxic, but it's not a foodstuff). So I kind of doubt it would be any other sort of rapeseed oil.
Unless "vegetable oil" in the UK doesn't have to be fit for human consumption? Is it something you'd use for, I don't know, thinning paint or something? --Trovatore (talk) 09:29, 6 June 2010 (UTC)[reply]
Indeed. In the U.S. "vegetable oil" tends to be primarily soybean oil, or a canola/soybean blend. Pure canola oil (labeled as such) tends to be slightly more expensive. I've also seen "vegetable oil" where the ingredient list was "one or more of soybean, corn or canola oil" (or something similar) implying that the mixture varied based on what was available cheap the day they bottled it. -- 140.142.20.229 (talk) 22:31, 5 June 2010 (UTC)[reply]
I have two bottles of vegetable oil in my kitchen - one Morrison's own brand, the other Lidl's own brand. Both are labelled as "vegetable oil", and both are 100% rapeseed oil. "Canola" is not a British English word. DuncanHill (talk) 23:47, 6 June 2010 (UTC)[reply]
(2) About 3 years ago I saw a television documentary about the vegetable oils used in food. They claimed that the cheapest vegetable oil is the palm oil. They also claimed that that was a bad thing because palm oil supposedly is far more likely to cause you diseases in the heart and blood vessels (cardiovascular disease) than any of the other vegetable oils. They even went as far as to point out (and warn against eating) specific food products who are made of palm oil, and who just say "vegetable oil" on the label.
--Seren-dipper (talk) 23:10, 6 June 2010 (UTC)[reply]
Unfortunately I do not have any references to eventual scientific research that would back up the following claim, and I do not remember any of the explanation about why, but I have it from a third hand source (a non scientific worker at a food safety authority) who claims she heard it at her workplace that the healthiest oil to sprinkle on your salad is sunflower seed oil, while the healtiest oil (the one that will turn least harmful) when exposed to high temperatures (i.e. fried or baked) is rape oil.
--Seren-dipper (talk) 23:39, 6 June 2010 (UTC)[reply]
That palm oil contains more saturated fats that sunflower oil etc seems to be the health issue eg [39] To complicate things there is an ongoing war in the literature - you find researchers from nigeria etc (places producing palm oil) pushin the health benefits and researchers from places that produce soya, sunflower oil etc saying it's not healthy.77.86.124.76 (talk) 00:15, 7 June 2010 (UTC)[reply]
Additionally there are different types of palm oil - with quite different constitutions [40] 77.86.124.76 (talk) 00:17, 7 June 2010 (UTC)[reply]

Standard Atmosphere...

This is a follow on from the highly informative 'boiling water' disscusion..

i) ' What is a standard 'atmosphere'  ?'

My very rough answer :
' A standard atmosphere, is a precisely determined combination of gases, which given that combination of gases has specfic definable physical properties, and obeys defined relationships between those propeties.'

However, this got me thinking, hence the following question :

ii) What factors need to be considered that might make an atmosphere non-standard

To which my initial thoughts where -

  • humidity (ie. 'humid' vs dry atmosphere) - and the 'liquid' component need not be water of course ;)
  • density of gas present.
  • liquid in proximity.
  • 'dust'
  • ionisation ?

Any thoughts from the Desk contributors? Sfan00 IMG (talk) 20:21, 5 June 2010 (UTC)[reply]

For physical measurements of bp the only factor that seems to be considered is pressure. see Standard conditions for temperature and pressure also Atmospheric_pressure#Standard_atmospheric_pressure and Atmosphere (unit) (some duplication of information)
As to definining what "standard air" is there's some links at Standard atmosphere, both U.S. Standard Atmosphere and International Standard Atmosphere are relevant.
In fact "one atmosphere" has been defined as both 100000Pa and 101325Pa (see first link)
The IUPAC doesn't seem to specify any other conditions other than pressure... There's definately at least two areas of definition - one for physics and another for atmospheric science.87.102.43.94 (talk) 20:38, 5 June 2010 (UTC)[reply]
The term I recall from high-school chemistry is not standard atmosphere (an atmosphere is a unit of pressure; no need for the standard), but standard temperature and pressure, abbreviated STP. If I recall correctly, STP was 25 degrees C and one atmosphere. Whereas my dad's chemistry books all seemed to refer things to 20 degrees C. I think I prefer that way. I find 25 Celsius (77 Fahrenheit) to be a little warm. -Trovatore (talk) 20:31, 6 June 2010 (UTC)[reply]

tobacco

during some wars they would drink pinecons because of coffee shortages. what did they smoke when there were tobacco shortages. —Preceding unsigned comment added by Alexsmith44 (talkcontribs) 21:16, 5 June 2010 (UTC)[reply]

According to our (fairly obscure) article Military_Administration_in_Belgium_and_North_France, sunflowers and jerusalem artichokes were used in the 40s. Nicotine, however, isn't present in either of those, so they would probably be rather unsatisfying. I tried to dig around on Google but couldn't find anything. When ersatz products weren't needed, cigarettes were frequently rationed. The Imperial War Museum has a picture of a tobacco tin containing tobacco substitute from the end of the first world war, although it doesn't describe its composition. Brammers (talk/c) 22:41, 5 June 2010 (UTC)[reply]
Hmmm. This rings a bell. I BELIEVE that there were certain plants (though I don't remember what they are) which would be -mixed- with tobacco, even in the smallest quantities. So you could have a cigarette with a small about of tobacco, and something else to bulk it out. I guess this gives you a little bit of the taste and nicotine. Hmmmm. SmokingNewton (MESSAGE ME) 00:59, 6 June 2010 (UTC)[reply]
A chap I knew who was in the navy in WWII said they smoked tea leaves when the tobacco ran out on board ship. He also said that this wasn't "very nice" but it was all they had. He told me that tea was always in abundance.--TrogWoolley (talk) 14:12, 6 June 2010 (UTC)[reply]
In the Second World War, British forces were issued with "Victory V" cigarettes. They were generally believed (because of their distinctive taste) to be made from a camel by-product. Some links here. DuncanHill (talk) 14:23, 6 June 2010 (UTC)[reply]

Can the Deepwater Horizon oil spill slow or even stop the ocean conveyor?

I realise that the complexities of the ocean conveyor are incredibly complex, and that any answers to this may be guesses at best. I read "Shutdown of thermohaline circulation" which varies from 5000 years (citatation needed) to just 2-3 years (especially from ice core data) for the Earth to swap to a full Ice Age. Scientific experiments currently point to a dramatic and exponential slowing of the conveyor, but again we don't know enough to say, "Well going by this in 7 years it will have stopped and we'll be in Ice Age".

But we do know that salinity in the water seems to drive it, and it's a very finely balanced system, which has numerous times in the past swapped state.

Given that we seem to be in the middle of such a swap, could the enormous amount of oil (which continues to rise with no current idea of how to stop it) stop the ocean conveyor? If it is so finely balanced and mere salinity of the water is extremely important, then wouldn't a Gulf full of crude oil be enough to stop the whole thing quickly? Could we be in ice age in 2010? I'll be very happy for any experts to shoot that idea down ..

blucat David —Preceding unsigned comment added by 198.142.19.12 (talk) 00:37, 6 June 2010 (UTC)[reply]

Prob'ly not -- the oil slick (as bad as it is) only covers a fairly small part of the Gulf's surface, so its effect on salinity is pretty insignificant. Also keep in mind that a large part of the oil is in underwater plumes, which have no effect on evaporation. Of course this is just a non-expert opinion from a reg'lar schmo who works at a refinery while taking evening classes at the university and writing adventure novels on weekends, and who knows a lot about oil refining but not much about ocean currents. FWiW 67.170.215.166 (talk) 01:57, 6 June 2010 (UTC)[reply]
The volume of the "enormous amount of oil spilled" in the "gulf full of crude oil" at the latest high limit for the leak rate of 19,000 barrels/day (USGS)is about what percent of the volume of water in the gulf, where a cubic meter=about 6.3 barrels of oil? I get about .000000006%. Edison (talk) 03:12, 6 June 2010 (UTC)[reply]

Plugging holes in ducts

I have a duct which conveys air under pressure. I have to create some holes in it for measurement. The holes will be larger than conventional ones - maybe 10 - 25mm in diameter. Can anyone suggest some easy ways of plugging them after I have finished? Thanks —Preceding unsigned comment added by 122.175.80.243 (talk) 04:27, 6 June 2010 (UTC)[reply]

What kind of pressure are we talking about? If it's fairly light pressure, duct tape on the inside of the duct might be easiest (maybe lain over two-inch cardboard or plastic circles for a bit of stiffness). duct tape around the outside might be sufficient as well, but it's ugly and not as strong. --Ludwigs2 04:44, 6 June 2010 (UTC)[reply]
As Ludwigs2 said we will need to know what sort of pressure the air is under before we can offer sensible answers, as depending on the pressure solutions will range from duct tape on the outside as mentioned above to things involving welding and steel plates. Another thing - is your only access to the inside of the duct through the holes, or can you reach the inside of the holes to e.g. apply patches? Equisetum (talk | email | contributions) 11:08, 6 June 2010 (UTC)[reply]
Duct tape can eventually fail. Metal tape with mastic like is used around flat roofs will deal with most problems with air conditioning ducts. For something this small just gluing something on it might do the job. Dmcq (talk) 12:35, 6 June 2010 (UTC)[reply]
Speed tape. In the UK I think it's just called "aluminium tape" (it's not just coated with aluminium, there's a Al foil as well) - it's definately more suitable for ducting that 'duck tape' especially in terms of gas resistance.77.86.124.76 (talk) 15:15, 6 June 2010 (UTC)[reply]
Corks or bungs? Works for champagne. 92.24.185.225 (talk) 13:48, 6 June 2010 (UTC)[reply]

Lee Smollin: author of "baby universe" book, adversary of multiverse? huh?

Lee Smolin wrote a book advocating (or maybe just describing the theory of?) "baby universes." Years later, in a celebrated comment, he said "This universe is the only one there is." I can't figure out if this means he abandoned the baby universe theory altogether, or if he was defining "universe" to mean "the universe that contains all the baby universes." Does anybody know how Smolin's two positions relate to each other? 63.17.74.45 (talk) 08:38, 6 June 2010 (UTC)[reply]

I don't, but note that one was written and another spoken, there's often quite a difference between what people are prepared to say and what they will write, my guess the the book was hypothesising.87.102.43.94 (talk) 13:39, 6 June 2010 (UTC)[reply]
They're both in print, the latter in this article (requires a login which you can get here). He evidently means "the universe that contains all the baby universes", making it a rather content-free statement as far as I can tell (though he seems to think otherwise). -- BenRG (talk) 19:32, 6 June 2010 (UTC)[reply]

BBC Report

I heard yesterday on the BBC Radio World Service that I was listening to while playing World of Warcraft, that the Cassini probe has discovered what could be life on Titan, due to certain gasses in the atmosphere being absorbed, or not reaching the serfice (surfice, sorry I have gone blank on the spelling???) This was anounced by NASA. I would like to read our article on this if we have one, can some one please link to it. Also, I have ready the article on Terraforming planets, and would like to read more in this field, can you please sugest articles in this vein. What university courses could one take to help further the future science of Terraforming, to help bring about this phenomenon faster, as I would love to be the man who 500 years from now is known as the father of Terraforming, or the Father of Human Colony II Mars. Thank you. —Preceding unsigned comment added by 62.172.58.82 (talk) 10:17, 6 June 2010 (UTC)[reply]

The featured article on Titan contains a lot of information, including cross-refs to the Cassini–Huygens mission and a section on Prebiotic conditions and possible life. We also have Colonization of Titan, and this article, which states that: "A widely published expert on terraforming, Christopher McKay, is also a co-investigator on the Huygens probe that landed on Titan in January 2005." Hope this helps - and good luck in getting there! Ghmyrtle (talk) 10:27, 6 June 2010 (UTC)[reply]
This is the story reported on the BBC: Hints of life found on Saturn moon --TammyMoet (talk) 10:44, 6 June 2010 (UTC)[reply]

Space travel

Given a timeframe of 5-10 years to create a vehicle, what is a reasonable quote for the amount of time it would take to send a probe to another solar system, given current technological levels? Ideally, the vehicle would return information to the earth about the solar system. I am assuming that current technology levels would not be good enough to reach relativistic speeds, so the perception in difference in time between the vehicle itself and the earth would be negligible (i.e., well within the margin of error for the quote). Magog the Ogre (talk) 16:14, 6 June 2010 (UTC)[reply]

Around 60,000 years. (I figure Cassini is currently at 25km/s, and 5 light years distance, is 60,000 years.) Cassini will slow down as it gets farther from the sun, but we could make some future ship a bit faster. But realistically we couldn't do it. The problem is how to decelerate when you reach the end point. We don't even know what the solar system looks like, we'd need artificial intelligence good enough to survey planets, then pick and calculate a gravity slow down, I'm not sure we are able to do that. Ariel. (talk) 17:10, 6 June 2010 (UTC)[reply]
That's hideous. Is there no manmade technology that could be put into a probe to quicken that span? Magog the Ogre (talk) 20:12, 6 June 2010 (UTC)[reply]
I expect the best method (using already existing technology) would be a kind of ion drive. The advantage of ion drives is that they can continue thrusting for a long time. If we want to launch in the next 10 years, though, it's still going to take at least hundreds of years. It would get there sooner if we waited until faster technology was invented and then launched. --Tango (talk) 20:37, 6 June 2010 (UTC)[reply]
There is no need to decelerate. It would be nice to do so, but you could still get lots of good science from a fly-by mission. I would expect our first interstellar missions to be fly-bys (just as our first interplanetary missions were). --Tango (talk) 20:37, 6 June 2010 (UTC)[reply]
I believe that if cost where not an issue, we could get a probe to the Alpha Centauri system in "just" a few hundred years. Ion thruster systems get better every year, (this report shows no hint of failure after more than 3 years at full power), and given their great mass/force ratio, they would be ideal for such a mission. Unfortunately I can't find the specs on the thrust they are capable of (which are quite small in comparison to chemical rockets), but even assuming just 10 km/s/year, (about 1/1000 the acceleration due to earth's gravity), the probe could achieve speeds more than 4 times that of the fastest current probes before it left the Solar System.-RunningOnBrains(talk) 21:01, 6 June 2010 (UTC)[reply]
It would be tough to power it. Assuming you ran for about 10 years, and the ship weighed 2 tons, you would need 10x10^12 joules, or about 32kW for those 10 years. Radioisotope thermoelectric generators have energies in the tens of watts, so that will never work. You would probably need a nuclear reactor such as Safe Affordable Fission Engine. But even then it would still take 15,000 years. Ariel. (talk) 01:41, 7 June 2010 (UTC)[reply]
In fact, I believe the biggest problem with this mission would not be getting there, but relaying what the probe sees back to earth. It would be useless to send a probe there if it can't send any information back, and New Horizons, after its fly-by of Pluto, is expected to take nine months to transmit just 8 GB of data back to Earth [41]. The amount of power needed to even reach those data transfer rates from 4 light years (and near the shine of a bright star)...well, I don't even want to begin to speculate.-RunningOnBrains(talk) 21:01, 6 June 2010 (UTC)[reply]
Let's speculate. New_Horizons#Telecommunications quotes 12W transmit power and 4.5 'light-hours' (signal latency). Link power is proportional to distance squared. Link power from Alpha Centauri would be 12W x sqr( 4.4 lightyears / 4.5 lighthours) = 1.1 kW Cuddlyable3 (talk) 21:38, 6 June 2010 (UTC)[reply]
A year is about 8000 hours, so multiplying by the square of the ratio (year/hr) should give you something like 700 MW, not 1.1 kW. Maybe you took the square root instead of the square? --Trovatore (talk) 21:50, 6 June 2010 (UTC)[reply]
Google says: 882 MW. 1.1kW is, indeed, what you get with a square root, not a square. --Tango (talk) 21:59, 6 June 2010 (UTC)[reply]
It occurs to me that one way to do better is to make the transmitting antenna bigger. With a bigger antenna you can get a more unidirectional signal — ultimately the power is still going to fall off as the square of the distance, but you can make the constant coefficient much more favorable.
How about having it shoot out some sort of very fine webbing a kilometer across, and using that as an antenna? Maybe it could even double as a light-sail. Obviously there are some practical challenges, but that's engineering's department; I'm over here in blue-sky big-picture territory. --Trovatore (talk) 22:10, 6 June 2010 (UTC)[reply]
The alternative is to use a laser. --Tango (talk) 22:30, 6 June 2010 (UTC)[reply]
Effectively that's like using a bigger antenna, because the important thing is the ratio of the aperture size to the wavelength (assuming spatial coherence across the entire aperture). But sure, a laser might be more practical than my idea. On the other hand it can't double as a light sail. So put together a presentation and present it at next week's meeting, please. --Trovatore (talk) 23:00, 6 June 2010 (UTC)[reply]
The Project Orion spaceship project was designed to reach 10% the speed of light, before it was scrapped. This would allow it, assuming constant maximum speed, to reach the Alpha Centauri system in 45 years. ~AH1(TCU) 01:27, 7 June 2010 (UTC)[reply]

How can using algae to make fuel be net energy positive?

Recently on the Science Channel some people like J. Craig Venter and Ray Kurzweil were talking about the prospects for the future of genetic engineering and one of them (I forget which one) was talking about making life that would more efficiently manufacture fuels (by which I presume he meant hydrocarbons) humans could use. The problem I don't get is, if you want a bacteria to produce for you a molecule of hydrocarbon that could release X joules of energy when you use it, wouldn't you have to feed that bacteria more than X joules to make it for you? So if you have X+ joules it would be more logical to store that up than use for your energy needs. 71.161.47.112 (talk) 16:55, 6 June 2010 (UTC)[reply]

See algae fuel - algae make fuel from sunlight - but different algaes make different types of fuel: some primarily carbohydrate/cellulose, others produce a substantial amount of oil as well. I'd guess they were suggesting making new types of algae that produce more oil as a percentage, since this is a more desirable form of fuel.77.86.124.76 (talk) 17:03, 6 June 2010 (UTC)[reply]
Algae are not bacteria (except blue-green algae which are, and should be called cyanobacteria), they are a lot like plants.77.86.124.76 (talk) 17:08, 6 June 2010 (UTC)[reply]
Comparing the first paragraph of Photosynthetic efficiency which says it's about 3-6% to http://en.wikipedia.org/wiki/File:PVeff%28rev100414%29.png , it seems like a battery with X joules charged by a solar cell is made off much less light than a container of oil of the same energy content made by algae. 71.161.47.112 (talk) 17:40, 6 June 2010 (UTC)[reply]
If we're talking about green algae then they usually are considered plants (not land plants obviously). Nil Einne (talk) 19:25, 6 June 2010 (UTC) [reply]
Yeah, but we're not talking about green algae; we're talking about blue-green algae, AKA cyanobacteria. Contrary to what 77.86.124.76 says, they are bacteria. However, in accordance with what 77.86.124.76 says, they are also "a lot like plants", in the sense that they use photosynthesis. --Trovatore (talk) 19:34, 6 June 2010 (UTC)[reply]
I tried to be careful in what I said (because I'm not a biologist and likely to make obvious schoolboy errors - as I did) - but all algae? Bacteria says they are unicellular, whereas Algae says it included multicellular forms. eg Are seaweed bacteria ?? please put me out of my misery77.86.124.76 (talk) 20:04, 6 June 2010 (UTC)[reply]
No, seaweed are not bacteria. But no one is proposing to make fuel out of seaweed. Well, as far as I know, anyway.
The problem is that the word algae comprises organisms that are more different among each other than you are from an earthworm. Green algae are plants; bluegreen algae are bacteria. You can't get too much more different than that.
Now, on reviewing the algae fuel article, it does look like I spoke a little hastily in assuming that all the fuel stock was to come from bluegreen algae. Apparently the fuel makers don't really care about the biology, just about getting the reduced carbon, so very small green algae are also OK with them. --Trovatore (talk) 20:11, 6 June 2010 (UTC)[reply]
It's ok I worked it out for myself, yes they use algae, in fact the article mentions as examples algae species, though cyanobacteria are potential biofuels too. [42] Anyway didn't you get the email - we're calling them cyanobacteria now to avoid just this confusion :) 77.86.124.76 (talk) 20:25, 6 June 2010 (UTC)[reply]

I reacted sodium acetate with yellow iron(III) chloride to form a blood-red solution. Is that iron(III) acetate? The article on iron(III) acetate says that it is insoluble, but in here it states that there is a coloration. My solution is soluble. Is it "nonbasic" iron(III) acetate as compared to basic iron(III) acetate, which is what the article is about? --Chemicalinterest (talk) 18:08, 6 June 2010 (UTC)[reply]

Yes. Boil it to get the oxy-acetate [43]
As for the article - it needs some corrective work - it definately should mention both acetates.77.86.124.76 (talk) 19:03, 6 June 2010 (UTC)[reply]
I took a picture of the acetate (
see right) and wanted to post it in, but it wasn't the right acetate that is described in the article. --Chemicalinterest (talk) 19:31, 6 June 2010 (UTC)[reply]
BTW, I did boil it and it fumed acetic acid and formed an insoluble orange-red chemical. --Chemicalinterest (talk) 19:33, 6 June 2010 (UTC)[reply]
I've tried to fix the article - it now mentions both compunds (Iron(III) acetate) 77.86.124.76 (talk) 19:58, 6 June 2010 (UTC)[reply]

chloroform mishap

I am trying to synthesis chloroform for a chemistry project and i ran into some trouble. the chemicals i combined were ethanol 80 proof calcium hypochlorate from pool tablets and a small amount of sodium bicarbonate to take care of the formic acid that should be made. anyways during the reaction i noticed prussa blue crystals forming at the bottom of the glass. any idea on what they could be? —Preceding unsigned comment added by 76.14.125.132 (talk) 22:02, 6 June 2010 (UTC)[reply]

I can't think of a reaction that would have made a blue colour off hand. Are you sure that the Calcium hypochlorite didn't have a blue additive in them - it seems some commercial formulations do eg [44] [45] 77.86.124.76 (talk) 23:52, 6 June 2010 (UTC)[reply]

Size of Rhinovirus

Hello, would anyone happen to know the size of a rhino virus (particularly the type which carries the common cold) ? I could not find anything on the Common Cold Article. Thanks!

~ QwerpQwertus ·_Talk_·_Contribs_· The Wiki Puzzle Piece Award 23:13, 6 June 2010 (UTC)[reply]

Approx 22-40nm, according to http://www.uq.edu.au/vdu/VDURhinovirus.htmHappymulletuk (talk) 23:23, 6 June 2010 (UTC)[reply]

Thanks!
~ QwerpQwertus ·_Talk_·_Contribs_· The Wiki Puzzle Piece Award 23:58, 6 June 2010 (UTC)[reply]

Private Moon base...

Let's assume I have a great amount of wealth (10's of billions of dollars). Is there anything to stop me contracting someone to build me a moon base?

What would the estimated cost of a self sufficient base be, including any systems needed to remain healthy on the surface for an adult lifetime (for example, detrimental effects due to reduced gravity).

Finally, (this depends on the answer), is there any individual or group with the funds to actually do this?

I look forward to your thoughts!!Happymulletuk (talk) 23:20, 6 June 2010 (UTC)[reply]

You would need hundreds of billions. Possibly even into the trillion dollar range to do that. Ariel. (talk) 23:43, 6 June 2010 (UTC)[reply]
Yes, such a thing would probably cost trillions (including matinence), though one may not even be possible with today's technology.
~ QwerpQwertus ·_Talk_·_Contribs_· The Wiki Puzzle Piece Award 00:04, 7 June 2010 (UTC)[reply]
"Self-sufficient" is probably beyond the range of current technology. To be truly self-sufficient, you'd need to be able to manufacture any spare part of any machine using only materials that can be mined from Moon rock. That's a very tall order. We might conceivably be able to extract water - and therefore oxygen and hydrogen+oxygen rocket fuel - but Moon rock points out that the only elements present in moon rock in appreciable quantities are Calcium, Aluminium, Silicon, Oxygen, Iron, Magnesium and Titanium. Without nitrogen or carbon (and a bunch of other stuff) - you are pretty fundamentally screwed when it comes to food production. You'd have to recycle every single gram of those two elements with fanatical attention to not losing anything whatever in the processing! You might manage to find deposits of carbon from meteorites that impacted the moon in the distant past - but nitrogen is harder.
As for funding, Forbes list of billionaires says that no single person could fund more than about $55 billion - and that's nowhere near enough. Constellation_Program#Cost says that the simplest manned mission would cost in the region of $200 billion. What you are proposing (a permenant self-sufficient base) would probably cost trillions. A consortium of the top 20 richest people could possibly establish a moon base - but there is no way they could keep it funded over the long term. So, I think the answer is a solid "No". SteveBaker (talk) 00:49, 7 June 2010 (UTC)[reply]

Avalanches

Can a strong gust of wind cause an avalanche? In particular, can an avalanche be triggered by the foehn winds that commonly occur in some mountain ranges (Alps, Rockies, etc.)? Thanks in advance! 67.170.215.166 (talk) 01:04, 7 June 2010 (UTC)[reply]

Since an avalanche can be triggered by a loud noise, it seems almost certain that they could be caused by strong winds -- but I can't see how it could ever be proven that any specific avalanche was caused by wind. Foehn winds tend to occur in specific weather conditions, so I don't see how you could disentangle the wind from the weather. Looie496 (talk) 01:22, 7 June 2010 (UTC)[reply]
Actually, avalanches caused by sound is an disproven fact (see Avalanche#Myths about avalanches). I have heard stories of low-flying helicopters causing avalanches, so I assume a strong windstorm could do it too. --The High Fin Sperm Whale 02:50, 7 June 2010 (UTC)[reply]

Lithium phosphide cell

The lithium/lithium phosphide cell looks interesting (two different patents ):

Questions:

  • [46] would this cell produce phosphine if it got wet? and would that prevent it ever being used (eg any comparative examples of potentially dangerous substances contained in a product)?
  • [47] a conducting allotrope of phosphorus is used - this suggests black or purple phosphorus - if the cell is discharged would this make polymeric polyphosphide ions , and if so would these ions react with water to produce phosphine or not? or would it be intercalation instead?
    • also an Intercalation (chemistry) reaction known anywhere between either Li or Li+ and conducting phosphorus?
    • also in the cell described above - I'd like to know whether the conducting phosphorus would be expected to reduced when in operation - are any polyphosphide anions known? In fact is anything much at all known about the surface chemistry or modification of black phosphorus?

Oh and does anyone know of any more? 77.86.124.76 (talk) 02:03, 7 June 2010 (UTC)[reply]

Elder people

Why elder people easily feels cold while the environment temperature is normal for other adults? roscoe_x (talk) 03:07, 7 June 2010 (UTC)[reply]

Two reasons may be - poor circulation, and/or lack of physical activity. There are other reasons too I think.77.86.124.76 (talk) 03:10, 7 June 2010 (UTC)[reply]
The OP may be interested in reading gerontology and geriatrics, which discuss the issues related to aging from slightly different perspectives. --Jayron32 04:27, 7 June 2010 (UTC)[reply]

Neural basis of learning

What are the neural substrates in goal directed learning? (Neurobiology of action). —Preceding unsigned comment added by Oxio-a (talkcontribs) 04:15, 7 June 2010 (UTC)[reply]

There are dozens of different goal-directed and goal-oriented pedagogy programs. You're going to have to be a bit more specific. If you are interested in neural substrates in general, see the article Cognitive neuroscience, which discusses them. --Jayron32 04:25, 7 June 2010 (UTC)[reply]

Well, within the context of contingency learning and extinction.

CNG engines and simple engines

Could you please tell me the difference between engines of vehicles using CNG(Compressed Natural Gas) as fuel and those using petrol/diesel as fuel. If there is no difference then why not such bikes are made that can use CNG as fuel? And even if there is difference does this difference only made it costly or undesirable to use CNG as fuel in bikes?--Myownid420 (talk) 04:18, 7 June 2010 (UTC)[reply]
See Compressed natural gas which discusses advantages and disadvantages of using CNG as a fuel, and Natural gas vehicle which discusses some about the vehicles themselves. They operate on a variety of different designs, so you'd want to see List of natural gas vehicles and select various models to see how they operate, and how they differ from "standard" gasoline or diesel vehicles. --Jayron32 04:23, 7 June 2010 (UTC)[reply]